Download as docx, pdf, or txt
Download as docx, pdf, or txt
You are on page 1of 78

Tuberculosis

1. True or False: Tuberculosis is a contagious bacterial infection caused by mycobacterium


tuberculosis and it only affects the lungs.
 True

 False

Answer: FALSE….tuberculosis is a contagious bacterial infection caused by mycobacterium


tuberculosis that affects the lungs AND other systems of the body like the joints, kidneys, brain,
spine, liver etc.

2. A 55-year old male patient is admitted with an active tuberculosis infection. The nurse will
place the patient in ___________________ precautions and will always wear
_____________________ when providing patient care?
 A. droplet, respirator

 B. airborne, respirator
 C. contact and airborne, surgical mask
 D. droplet, surgical mask
The answer is B. A patient with ACTIVE TB is contagious. The bacterium, mycobacterium
tuberculosis which causes TB, is so small that it can stay suspended in the air for hours to days.
Therefore, the nurse will place the patient in AIRBORNE precautions. In addition, a special
mask must be worn called a respirator (also referred to as an N95 mask…..a surgical mask does
NOT work with this condition).

3. Which statement is correct regarding mycobacterium tuberculosis?


 A. This bacterium is an anaerobic type of bacteria.
 B. It is an alkali bacterium that stains bright red during an acid-fast smear test.

 C. It is known as being an aerobic type of bacteria.


 D. It’s an acid-fact bacterium that stains bright green during an acid-fast smear test.
The answer is C. Mycobacterium tuberculosis is AEROBIC (it thrives in conditions that are high
in oxygen), and it is an ACID-FAST bacterium, which means when it is stained during an acid-
fast smear it will turn BRIGHT RED.

4. Your patient with a diagnosis of latent tuberculosis infection needs a bronchoscopy. During
transport to endoscopy, the patient will need to wear?
 A. N95 mask
 B. Surgical mask

 C. No special PPE is needed


 D. Face mask with shield
The answer is C. Patients with a latent tuberculosis infection are NOT contagious. Therefore, no
special PPE is needed for the patient during transport. HOWEVER, if the patient had ACTIVE
tuberculosis they would need to wear a surgical mask during transport.

5. You are assessing your newly admitted patients who are all presenting with atypical signs and
symptoms of a possible lung infection. The physician suspects tuberculosis. So, therefore, the
patients are being monitored and tested for the disease. Select all the risk factors below that
increases a patient’s risk for developing tuberculosis:
 A. Diabetes
 B. Liver failure

 C. Long-term care resident

 D. Inmate

 E. IV drug user

 F. HIV
 G. U.S. resident
The answers are C, D, E, and F. Remember from our lecture we discussed the risk factors for
developing TB and to remember them I said remember the mnemonic “TB Risk”. It stands for
tight living quarters (LTC resident, prison, homeless shelter etc.), below or at the poverty line
(homeless), refugee (especially in high risk countries), immune system issue such as HIV,
substance abusers (IV drugs or alcohol), Kids less than the age of 5….all these are risk factors.

6. Your patient is diagnosed with a latent tuberculosis infection. Select all the correct statements
that reflect this condition:
 A. “The patient will not need treatment unless it progresses to an active
tuberculosis infection.”

 B. “The patient is not contagious and will have no signs and symptoms.”

 C. “The patient will have a positive tuberculin skin test or IGRA test.
 D. “The patient will have an abnormal chest x-ray.”
 E. “The patient’s sputum will test positive for mycobacterium tuberculosis.”
The answers are B and C. The patient WILL need medical treatment to prevent this case of LBTI
from developing into an active TB infection later on. The patient will NOT have an abnormal
chest x-ray or a positive sputum test. This is only in active TB.

7. A 52-year old female patient is receiving medical treatment for a possible tuberculosis
infection. The patient is a U.S. resident but grew-up in a foreign country. She reports that as a
child she received the BCG vaccine (bacille Calmette-Guerin vaccine). Which physician’s order
below would require the nurse to ask the doctor for an order clarification?

 A. PPD (Mantoux test)


 B. Chest X-ray
 C. QuantiFERON-TB Gold (QFT)
 D. Sputum culture
The answer is A. Patients who have received the BCG vaccine will have a false positive on a
PPD (Mantoux test), which is the tuberculin skin test. The BCG vaccine is a vaccine to prevent
TB. It is given in foreign countries to children to prevent TB. Therefore, the person has already
been exposed to the bacteria via vaccine and will have a false positive. A QuantiFERON-TB
Gold test is a better option for this patient. It is a blood test.

8. You’re teaching a group of long-term care health givers about the signs and symptoms of
tuberculosis. What signs and symptoms will you include in your education?
 A. Cough for a minimum of 6 weeks

 B. Night sweats
 C. Weight gain

 D. Hemoptysis

 E. Chills

 F. Fever

 G. Chest pain
The answers are B, D, E, F, and G. Option A is wrong because a cough should be present for 3
weeks or more (NOT 6 weeks). Option C is wrong because the patient will experience weight
LOSS (not gain).

9. A patient has a positive PPD skin test that shows an 8 mm induration. As the nurse you know
that:
 A. The patient will need to immediately be placed in droplet precautions and
started on a medication regime.
 B. The patient will need a chest x-ray and sputum culture to confirm the test results

before treatment is provided.


 C. The patient will need an IGRA test to help differentiate between a latent
tuberculosis infection versus an active tuberculosis infection.
 D. The patient will need to repeat the skin test in 48-72 hours to confirm the
results.
The answer is B. A positive PPD result does NOT necessarily mean the patient has an active
infection of TB. The patient will need a chest x-ray and sputum culture to determine if
mycobacterium tuberculosis is present and then treatment will be based on those results. The
IGRA test does NOT differentiate between LTBI or an active TB infection. Patients are placed in
airborne precautions (NOT droplet) if they have ACTIVE TB.

10. A patient has a PPD skin test (Mantoux test). As the nurse you tell the patient to report back
to the office in _________ so the results can be interpreted?
 A. 24-48 hours
 B. 12-24 hours

 C. 48-72 hours
 D. 24-72 hours
The answer is C. The patient should report back in 48-72 hours. If they fail to, the test must be
repeated.

11. A 48-year old homeless man, who is living in a local homeless shelter and is an IV drug user,
has arrived to the clinic to have his PPD skin test assessed. What is considered a positive result?
 A. 5 mm induration
 B. 15 mm induration
 C. 9 mm induration

 D. 10 mm induration
The answer is D. 15 mm induration is positive in ALL people regardless of health history or risk
factors. However, for patients who are homeless (living in homeless shelter) and are IV drug
users, a 10 mm or more is considered positive.
12. The physician orders an acid-fast bacilli sputum culture smear on a patient with possible
tuberculosis. How will you collect this?
 A. Collect 2 different sputum specimens 12 hours apart
 B. Collect 3 different sputum specimens (one in the morning, afternoon, and at
night)

 C. Collect 3 different sputum specimens on 3 different days


 D. Collect 2 different sputum specimens on 2 different days
The answer is C. This is how an AFB sputum culture is collected.

13. A patient receiving medical treatment for an active tuberculosis infection asks when she can
starting going out in public again. You respond that she is no longer contagious when:

 A. She has 3 negative sputum cultures

 B. Her signs and symptoms improve


 C. She has completed the full medication regime
 D. Her chest x-ray is normal

 E. She has been on tuberculosis medications for about 3 weeks


The answers are A, B, and E. These are all criteria for when a patient with active TB can return
to public life (school, work, running errands). Until then they are still contagious and must stay
home in isolation.

14. As the nurse you know that one of the reasons for an increase in multi-drug-resistant
tuberculosis is:
 A. Incorrect medication ordered
 B. Increase in tuberculosis cases nationwide
 C. Incorrect route of drug ordered
 D. Noncompliance due to duration of medication treatment needed
The answer is D. Patients must be on medication treatment for about 6-12 months (depending on
the type of TB the patient has). This leads to noncompliant issues. DOT (directly observed
therapy) is now being instituted so compliance is increased. This is where a public health nurse
or a trained DOT worker will deliver the medication and watch the patient swallow the pill until
treatment is complete.

15. Your patient, who is receiving Pyrazinamide, report stiffness and extreme pain in the right
big toe. The site is extremely red, swollen, and warm. You notify the physician and as the nurse
you anticipated the doctor will order?
 A. Calcium level
 B. Vitamin B6 level

 C. Uric acid level


 D. Amylase level
The answer is C. This medication can increase uric acid levels which can lead to gout. The
patient’s signs and symptoms are classic findings in a gout attack.

16. You note your patient’s sweat and urine is orange. You reassure the patient and educate him
that which medication below is causing this finding?
 A. Ethambutol
 B. Streptomycin
 C. Isoniazid

 D. Rifampin
The answer is D. This medication will cause body fluids to turn orange.
17. A patient with active tuberculosis is taking Ethambutol. As the nurse you make it priority to
assess the patient’s?
 A. hearing
 B. mental status
 C. vitamin B6 level

 D. vision
The answer is D. This medication can cause inflammation of the optic nerve. Therefore, it is very
important the nurse asks the patient about their vision. If the patient has blurred vision or reports
a change in colors, the MD must be notified immediately.

18. A patient taking Isoniazid (INH) should be monitored for what deficiency?
 A. Vitamin C
 B. Calcium

 C. Vitamin B6
 D. Potassium
The answer is C. This medication can lead to low Vitamin B6 levels. Most patients will take a
supplement of B6 while taking this medication.

19. A patient is taking Streptomycin. Which finding below requires the nurse to notify the
physician?
 A. Patient reports a change in vision.
 B. Patient reports a metallic taste in the mouth.

 C. The patient has ringing in their ears.


 D. The patient has a persistent dry cough.
The answer is C. This medication can be very toxic to the ears (cranial nerve 8). Therefore, it is
alarming if the patient reports ringing in their ears, which could represent ototoxicity.
Appendicitis
1. True or False: The appendix is found on the left lower side of the abdomen and is connected to
the cecum of the large intestine.
 True

 False
The answer is FALSE: The appendix is found on the RIGHT (not left) lower side of the abdomen
and is connected to the cecum of the large intestine.

2. Select all the following options that are NOT causes of appendicitis:
 A. Fecalith

 B. Routine usage of NSAIDs

 C. Infection due to Helicobacter pylori


 D. Lymph node enlargement due to viral or bacterial infection

 E. Diet low in fiber


The answers are: B, C, and E. These options are NOT causes of appendicitis. Routine usage of
NSAIDS and infection due H. pylori are causes of peptic ulcers. While a diet low in fiber is
thought to be the cause of diverticulosis. Fecalith and lymph node enlargement due to viral or
bacterial infection (such as mononucleosis etc.) can cause appendicitis.

3. A 23 year old patient is admitted with suspected appendicitis. The patient states he is having
pain around the umbilicus that extends into the lower part of his abdomen. In addition, he says
that the pain is worst on the right lower quadrant. The patient points to his abdomen at a location
which is about a one-third distance between the anterior superior iliac spine and umbilicus. This
area is known as what?
 A. Rovsing's Point
 B. Hamman's Point

 C. McBurney's Point
 D. Murphy's Point
The answer is C. This is known as McBurney's Point and is a classic sign and symptom in
patients with appendicitis.

4. Thinking back to the scenario in question 3, what other signs and symptoms are associated
with appendicitis. SELECT-ALL-THAT-APPLY:
 A. Increased red blood Cells
 B. Patient has the desire to be positioned in the prone position to relieve pain

 C. Umbilical pain that extends in the right lower quadrant

 D. Abdominal rebound tenderness


 E. Abdominal Flaccidity
The answers are: C and D. These are classic signs and symptoms found in patients with
appendicitis. Option A is wrong because the patient may have increased WHITE blood cells (not
red). Option B is wrong because the patient may have the desire to be in the fetal position (side-
lying with the knees bent) to relieve the pain. The prone position would increase the pain. Option
E is wrong because the patient would have abdominal RIGIDITY (not flaccidity).

5. An 18 year old patient is admitted with appendicitis. Which statement by the patient requires
immediate nursing intervention?
 A. "The pain hurts so much it is making me nauseous."
 B. "I have no appetite."

 C. "The pain seems to be gone now."


 D. "If I position myself on my right side, it makes the pain less intense."
The answer is C. It is important that the nurse monitors the patient's pain level. If the patient
reports that the pain has suddenly decreased or is gone, this is a warning sign that the appendix
may have perforated (ruptured). If the appendix has ruptured, the sudden decrease in pain will be
followed by more pain due to peritonitis (which is life- threatening). Therefore, the nurse should
notify the doctor immediately.

6. You're providing education to a group of nursing students about the care of a patient with
appendicitis. Which statement by a nursing student requires re-education about your teaching?
 A. "After an appendectomy the patient may have a nasogastric tube to remove
stomach fluids and swallowed air."
 B. "Non-pharmacological techniques for a patient with appendicitis include

application of heat to the abdomen and the side-lying position."


 C. "The nurse should monitor the patient for signs and symptoms of peritonitis
which includes increased heart rate, respirations, temperature, abdominal
distention, and intense abdominal pain."
 D. "It is normal for some patients to have shoulder pain after a laparoscopic
appendectomy.”
The answer is B. This statement by the nursing student requires re-education because heat should
NEVER be applied to abdomen if appendicitis is suspected or known. Heat application can
increase the risk of appendix perforation. Ice application is recommended, if warranted.
However, the side-lying position can help relieve the patient’s pain and is recommended. All the
other options are correct.

7. Your patient is 4 days post-opt from an appendectomy. Which assessment finding requires
further evaluation?

 A. The patient reports their last bowel movement was the day before surgery.
 B. The patient reports incisional pain.
 C. The patient coughs and deep breathes while splinting the abdominal incision.
 D. Options A and C
The answer is A. If the patient has not had a bowel movement 2-3 days after surgery it requires
further evaluation such as listening to bowel sounds and asking the patient if they are passing
gas. If the patient has no bowel sounds or does NOT report passing gas, the doctor should be
notified. Options B and C are normal findings.

8. A patient is recovering after having an appendectomy. The patient is 48 hours post-opt from
surgery and is tolerating full liquids. The physician orders for the patient to try solid foods. What
types of foods should the patient incorporate in their diet?

 A. Foods high in fiber


 B. Foods low in fiber
 C. Foods high in carbohydrates
 D. Foods low in protein
The answer is A. It is best for the patient to follow a diet high in fiber to prevent straining during
bowel movements.

9. A patient is scheduled for appendectomy at noon. While performing your morning assessment,
you note that the patient has a fever of 103.8 'F and rates abdominal pain 9 on 1-10. In addition,
the abdomen is distended and the patient states, "I was feeling better last night but it seems the
pain has become worst." The patient is having tachycardia and tachypnea. Based on the scenario,
what do you suspect the patient is experiencing?
 A. Pulmonary embolism
 B. Colon Fistulae

 C. Peritonitis
 D. Hemorrhage
The answer is C. Based on the patient's presenting symptoms, the patient is most likely
experiencing peritonitis because the appendix has ruptured. The key clues in this scenario are the
classic signs and symptoms of peritonitis (tachycardia, tachypnea, high temperature, and
abdominal pain/distension) along with the patient's statement that they were feeling better last
night (hence probably the time the appendix ruptured) which periodically relieved the pain at the
appendix but allowed for the contents of the appendix to leak into the peritoneal cavity....hence
causing peritonitis.

Pneumonia
1. A patient is presenting with mild symptoms of pneumonia. The doctor diagnoses the patient
with "walking pneumonia". From your nursing knowledge, you know this type of pneumonia is
caused by what type of infectious agent?
 A. Fungi
 B. Streptococcus pneumoniae

 C. Mycoplasma pneumoniae
 D. Influenza
The answer is C.

2. A patient was admitted to the intensive care unit 48 hours ago for treatment of a gunshot
wound. The patient has recently developed a productive cough and a fever of 104.3 'F. The
patient is breathing on their own and doesn't require mechanical ventilation. On assessment, you
note coarse crackles in the right lower lobe. A chest x-ray shows infiltrates with consolidation in
the right lower lobe. Based on this specific patient scenario, this is known as what type of
pneumonia?
 A. Aspiration pneumonia
 B. Ventilator acquired pneumonia

 C. Hospital-acquired pneumonia
 D. Community-acquired pneumonia
The answer is C. The key words to let you know this is hospital-acquired pneumonia and NOT
community-acquired is that the patient was admitted with a gunshot wound AND has been
hospitalized for 48 hours. If the patient presents with signs and symptoms of pneumonia 48-72
hours after admission it is classified as hospital-acquired. This is not ventilator acquired because
the patient is not on mechanical ventilation and there is nothing in the scenario that leads us to
think it is aspiration pneumonia.

3. Which of the following patients are MOST at risk for developing pneumonia? Select-all-that-
apply:

 A. A 53 year old female recovering from abdominal surgery.


 B. A 69 year old patient who recently received the pneumococcal conjugate
vaccine.
 C. A 42 year old male with COPD and is on continuous oxygen via nasal

cannula.

 D. A 8 month old with RSV (respiratory syncytial virus) infection.


The answers are A, C, and D. Risks factors for pneumonia include: recent surgery, lung disorder
(ex: COPD), and viral infection (ex: RSV). Option B is a preventive measure in preventing
pneumonia.

4. You're caring for a patient with pneumonia. The patient has just started treatment for
pneumonia and is still experiencing hypoxemia. You know that respiratory acidosis is very
common with patients with pneumonia. Which arterial blood gas readings below represent
respiratory acidosis that is NOT compensated?
 A. pH 7.29, PaCO2 55, HCO3 23, PO2 85
 B. pH 7.48, PaCO2 35, HCO3 22, PO2 85

 C. pH 7.20, PaCO2 20, HCO3 28, PO2 85


 D. pH 7.55, PaCO 63, HCO3 19, PO2 85
The answer is A. This is the only ABG result that represents respiratory acidosis. If you are
unfamiliar with how to interpret ABGs, please read and review this article that explains how to
do this: https://www.registerednursern.com/respiratory-acidosis-nclex-review-notes-with-
mnemonic-quiz-acid-base-imbalance-notes/
5. Which of the following are typical signs and symptoms of pneumonia? Select-all-that-apply:
 A. Stridor

 B. Coarse crackles

 C. Oxygen saturation less than 90%


 D. Non-productive, nagging cough

 E. Elevated white blood cells


 F. Low PCO2 of less than 35

 G. Tachypnea
The answers are B, C, E, and G. These are typical signs and symptoms of pneumonia. Stridor is
not very common. A PRODUCTIVE cough that can be nagging is very typical, and there is
usually a HIGH PCO2 of 45 or greater due to the lungs retaining carbon dioxide.

6. You're educating a patient with pneumonia on how to deep breathe by using an incentive
spirometer. Which of the following is the correct way to use this device?
 A. Encourage the patient to use it twice a day.
 B. The patient exhales into the device rapidly and then coughs.
 C. The patient inhales slowly from the device until no longer able, and then holds

breath for 6 seconds and exhales.


 D. The patient rapidly inhales 10 times from the device and then exhales for 6
seconds.
The answer is C. The patient will inhale slowly from the device until no longer able, and then
hold breath for 6 seconds and exhale. The patient should use the device at least 10 times every 1-
2 hours while awake. The other options are incorrect ways to use the device. For a video
demonstration on how to use an incentive spirometer, watch this video:
https://www.youtube.com/watch?v=pZxq6oroGhk
7. A 72 year-old male patient who is diagnosed with bilateral lower lobe pneumonia is admitted
to your unit. The patient has a history of systolic heart failure and arthritis. On assessment, you
note the patient has a respiratory rate of 21, oxygen saturation 93% on 2L nasal cannula, is alert
& oriented, and has a productive cough with green/yellowish sputum. Which of the following
nursing interventions will you provide to this patient based on your assessment findings and the
patient's diagnosis? Select-all-that-apply:
 A. Keep head-of-the-bed less than 30 degrees at all times.

 B. Collect sputum cultures.


 C. Encourage 3L of fluids a day to keep secretions thin.

 D. Encourage incentive spirometer usage

 E. Provide education about receiving the Pneumovax vaccine every 5 years.


The answers are B, D, and E. You would not keep the head-of-bed less than 30 degrees because
this impedes breathing and increases the risk of aspiration. Also, since the patient has systolic
heart failure you would NOT encourage 3L of fluids per day because the patient would not
tolerate the extra fluid. However, in normal situations (if a patient does not have heart failure
etc.) you would want to encourage fluids to keep secretions thin.

8. You’re providing discharge teaching to a patient who was admitted for pneumonia. You are
discussing measures the patient can take to prevent pneumonia. Which of the following
statements by the patient indicates they did NOT understand your education material?
 A. "I’ll use hand sanitizer regularly while I'm out in public."
 B. "It is important I don’t receive the Pneumovax vaccine since I'm already

immune to pneumonia."
 C. "I will try to avoid large crowds of people during the peak of flu season."
 D. "It is important I try to quit smoking."
The answer is B. All the other options are correct statements regarding the prevention of
pneumonia. However, option B is incorrect because although the patient has had pneumonia they
should still receive the Pneumovax vaccine to prevent other forms of pneumonia.

9. A patient is admitted with pneumonia. Sputum cultures show that the patient is infected with a
gram positive bacterium. The patient is allergic to Penicillin. Which medication would the
patient most likely be prescribed?

 A. Macrolide
 B. Cephalosporins
 C. Pencillin G
 D. Tamiflu
The answer is A. Marcolides are used to treat gram-positive bacterial infections and are used in
patients with penicillin allergies. Penicillin G is a penicillin antibiotic and would not be used
because of the patient's allergy. In addition, usually if a patient is allergic to penicillin there is an
increased chance they are allergic to cephalosporin....therefore it would not be used as well.

10. A 25 year-old female patient with pneumonia is prescribed Doxycycline. What question is
important to ask the patient prior to administration of this medication?

 A. "Do you take birth control pills?"


 B. "Are you allergic to Penicillin?"
 C. "Are you allergic to eggs?"
 D. "Do you have a history of diabetes?"
The answer is A. Doxycycline is part of the Tetracycline antibiotic family. This medication can
decrease the effectiveness of birth control pills. Since the patient is female and within child-
bearing age, the question should be asked if she is taking birth control pills so you can educate
her on using another form of birth control to prevent pregnancy.
11. Select all the medications used to treat pneumonia that are narrow-spectrum?

 A. Macrolides
 B. Tamiflu
 C. Fluroquinolones

 D. Penicillins
The answers are A and D. These are the only options that are considered narrow spectrum
antibiotics which means they target specific family of bacteria.

12. A patient is admitted with rupture of the Achilles tendon. The patient was recently treated
with antibiotics for pneumonia. Which of the following medications below can cause this
adverse effect?
 A. Penicillin

 B. Fluroquinolones
 C. Tetracyclines
 D. Macrolides
The answer is B. Fluroquinolones can cause tendon rupture along with c.diff infection and
cardiac arrhythmia such as prolonged QT interval.

13. A patient is being discharged home on Doxycyline for treatment of pneumonia. Which
statement by the patient indicates they understood your education material?

 A. "I will wear sunscreen when outdoors."


 B. "I will avoid green leafy vegetables while taking this medication."
 C. "I will monitor my blood glucose regularly due to the side effects of
hypoglycemia."
 D. "I will take this medication with a full glass of milk."
The answer is A. Doxycycline increases the skin's sensitivity to the sun, so the patient should
wear sunscreen when outdoors. Also, the patient should not take this medication with antacids or
milk products because this affects the absorption of the medication. Options B and C are
incorrect statements.

14. You are about to hang a bag of intravenous Vancomycin for a patient who has severe
pneumonia. Which statement by the patient causes you to hold the bag of Vancomycin and notify
the doctor immediately?
 A. "I'm seeing yellow halos around the light."
 B. "My mouth tastes like metal."
 C. "My head hurts."

 D. "I have this constant ringing in my ears."


The answer is D. Vancomycin can cause ototoxicity. Roaring or ringing in the ears are a possible
sign/symptom of this adverse effect. Option A happens with Digoxin toxicity. Options B and C
are vague and are not a common adverse reaction to Vancomycin.
Cov.
1. Which statement about the coronavirus is INCORRECT?
 A. "Coronavirus is a term used to describe a large group of viruses that cause
respiratory symptoms."
 B. "MERS, SARS, and COVID-19 are types of coronaviruses."

 C. "Coronaviruses are rarely zoonotic in transmission."


 D. "The coronavirus is contagious."
The answer is C. Coronaviruses ARE zoonotic in transmission. This means that they are spread
from animals to humans. Once this happens the virus can be spread from human to human. All
the other statements are true about the coronavirus.

2. When a virus is transmitted from an animal to a human this is termed as?


 A. mammalian event

 B. zoonotic spillover event


 C. interspecies transmission event
 D. intraspecies spillover event
The answer is B. A zoonotic spillover event is when a virus is transmitted from an animal
carrying the virus to a human. This is rare but can happen as we have seen with the
coronaviruses.

3. Middle East Respiratory Syndrome (MERS) started in ______________ and was transmitted
from _________ to humans.
 A. China; civet cat
 B. Asia; seafood

 C. Saudi Arabia; camels


 D. China; pangolin
The answer is C. MERS started in Saudi Arabia and was transmitted from camels to humans.

4. Severe Acute Respiratory Syndrome (SARS) started in ______________ and was transmitted
from _________ to humans.

 A. China, civet cat


 B. China, pangolin
 C. Saudi Arabia, camels
 D. Asia; seafood
The answer is A. Severe Acute Respiratory Syndrome (SARS) started in China and was
transmitted from civet cats to humans.

5. COVID-19 started in what country?

 A. Wuhan, China
 B. Macau, China
 C. Guangzhous, China
 D. Beijing, China
The answer is A: Wuhan, China

6. COVID-19 is genetically similar to what other coronavirus but it's actually a different virus?
 A. MERS

 B. SARS
The answer is B. COVID-19 has been found to be genetically similar to SARS but it's a different
virus.

7. What is the name of the virus that causes COVID-19?


 A. SARS-CoV-1
 B. HCoV-Co43
 C. MERS-CoV

 D. SARS-CoV-2
The answer is D: SARS-CoV-2 (severe acute respiratory syndrome coronavirus 2 ). COVID-19
is genetically similar to the SARS coronavirus BUT it is a different virus.

8. How is COVID-19 PRIMARILY spread to others?


 A. Airborne

 B. Droplet
 C. Blood
 D. Fecal-oral
The answer is B. COVID-19 is primarily spread to others via respiratory droplets. However, it
may be spread via contact to the eyes, noses, or mouth since the virus may be able to live on
surface. More research is needed at this time to investigate other routes of transmission.
9. Select the signs and symptoms of COVID-19:

 A. Fever

 B. Cough

 C. Sore throat

 D. Fatigue

 E. Shortness of breath
All the answers are signs and symptoms of COVID-19.

10. What are some preventive measures the nurse can teach the members of the community on
how to prevent acquiring COVID-19? Select all that apply:

 A. Handwashing

 B. Wearing a facemask when going around others

 C. Covering mouth and nose when sneezing and coughing

 D. Avoiding others who are sick

 E. Staying home when sick


 F. Social distancing at a distance of 2 feet
 G. Avoid eating to-go foods
The answers are A, B, C, D, E. These are recommendations the nurse should teach the patient.
Option F is wrong because the person should distance at least 6 feet, and option G is wrong
because it is okay to get foods to-go.
11. TRUE or FALSE: A person with COVID-19 can transmit the virus even though they are
asymptomatic.

 True
 False
The answer is TRUE

HIV (AIDS)
1. The Human Immunodeficiency Virus (HIV) mainly attacks what type of cells in the human
body?
 A. Red Blood Cells

 B. CD4 positive cells


 C. Stem Cells
 D. Platelets
The answer is B. The HIV virus attacks the human body's immune system, specifically the CD4
positive cells...mainly the helper t cells. These cells are white blood cells that help the immune
system fight infection.

2. The Human Immunodeficiency Virus (HIV) can NOT be spread in what type of fluid below?
Select all that apply:
 A. Breastmilk
 B. Blood

 C. Tears
 D. Semen
 E. Vaginal Fluid

 F. Sweat
The answers are C and F. HIV can NOT be spread in tears or sweat (unless blood is present
which rarely occurs). HIV is spread in the following fluids: breastmilk, blood, semen, and
vaginal fluid.
3. Which statement below is not true regarding the role of the helper t cell?
 A. The helper T cell releases cytokines to help activate other immune system cells.
 B. The helper T cell is part of the adaptive immune system.

 C. The helper T cell is cytotoxic and kills invaders.


 D. The helper T cell has CD4 receptors found on its surface.
The answer is C. All the other options are true statements about the helper T cell. Option C is
NOT true. Helper T cells are not cytotoxic and kill invaders (this is another type of t cells called
cytotoxic t cell). Helper T cells are “helpers” in that they HELP the immune system by releasing
cytokines, which help activate other immune system cells.

4. Identify the correct statements about the anatomy of the Human Immunodeficiency Virus
(HIV). Select all that apply:

 A. HIV is a retrovirus.
 B. Inside the virus is packaged DNA.
 C. The protein projections found on the virus’ surface play a key role in attaching

to the receptors on the helper t-cell.


 D. The glycoproteins (specifically GP140) are vital for engaging the receptors on
the targeted cell.
The answers are A and C. Options B is an incorrect statement, it should say “Inside the virus is
packaged RNA (not DNA)”, and Option C is incorrect because it should say “The glycoproteins
(specifically GP120…NOT GP140) are vital for engaging the CD4 receptor on the targeted cell.

5. The first step in the “Lifecycle of HIV” is the attachment step. During this step what co-
receptors are also targeted so entry can be gained inside the cell?

 A. CCR5 or CXCR4
 B. CD4 or CD2
 C. CCR4 or CXCR5
 D. CD8 or CD5
The answer is A: CCR5 or CXCR4. Attachment occurs when the glycoproteins (GP120)
projections make contact and bind with a CD4 receptor. In addition, there is also binding with
certain co-receptors called CCR5 or CXCR4 to gain entry into the cell.

6. What step in the ““Lifecycle of HIV” does the Human Immunodeficiency Virus (HIV) become
united with the targeted cell and dumps its contents into that cell?
 A. Integration

 B. Fusion
 C. Budding
 D. Assembly
The answer is B: Fusion

7. Which statement below best describes the role of reverse transcriptase?


 A. It’s an enzyme that helps cut up the long protein chains of HIV.
 B. It plays a key role in the maturity of the virus.

 C. Reverse transcriptase is an enzyme that turns viral RNA into viral DNA.
 D. Reverse transcriptase is an enzyme that allows the viral DNA to become part of
the cell’s DNA.
The answer is C: Reverse transcriptase is an enzyme that turns viral RNA into viral DNA.

8. Fill in the blank: Once inside the nucleus of the cell, the Human Immunodeficiency Virus
(HIV) needs to become part of the cell’s DNA. To do this, the virus releases an enzyme called
__________________.
 A. Protease
 B. Reverse transcriptase
 C. Kinase

 D. Integrase
The answer is D: Integrase

9. What enzyme is responsible for cutting long chains of virus to help prep the HIV for maturity?
 A. Integrase

 B. Protease
 C. Reverse transcriptase
 D. Kinase
The answer is B: Protease

10. A 30-year-old patient is in the Acute Stage of HIV. What findings below correlate with this
stage of HIV? Select all that apply:
 A. CD4 level <500 cells/mm3

 B. No present of Opportunistic Infections

 C. High viral load

 D. Patient reports flu-like symptoms


 E. Patient is asymptomatic
The answers are B, C, and D. The Acute Stage of HIV is the first stage and tends to occur a
couple of weeks to a month after becoming infected. The patient’s viral load is very HIGH
during this time, but the CD4 count should be greater than 500 cells/mm3. Therefore, NO
opportunistic infections are present during this time (the CD4 count is high enough to fight off
these types of infections/diseases). In addition, the patient may report flu-like symptoms (aches,
joint pain, headache, fever, fatigue, sore throat, swollen lymph nodes, GI upset, and rash). The
patient is usually asymptomatic in the 2nd stage (Chronic Stage of HIV).
11. A patient arrives to the clinic and requests an HIV test. The patient had unprotected sexual
intercourse 2 days ago with a person who may have HIV. As the nurse you know there is a
window period for detecting an infection of HIV. What statements should you provide to the
patient about this window period and testing for HIV? Select all that apply:

 A. No test is available at this time to show immediate infection.


 B. The window period is the time when you become infected with HIV to when a

test can deliver positive results.

 C. Window periods vary depending on the type of HIV test administered.


 D. The absolute earliest an HIV test can detect HIV is about 3 months.
The answers are A, B, C. These are correct statement. Option D is incorrect because the NAT
(nucleic acid test) can detect HIV the earliest of all the test types. It can detect around 10 days
after exposure for some patients. It assesses for the virus’ genetic material and measures the
amount of virus present in the blood. It is not commonly ordered and is used only for high risk
patients.

12. The physician orders a combination HIV antigen/antibody test on a patient. The patient was
potentially exposed to HIV 3 weeks ago. What HIV antigen does this test assess for?
 A. GP120

 B. p24
 C. GP41
 D. P35
The answer is B: p24

13. Which HIV test can give the earliest test results?
 A. Nucleic Acid Test (NAT)
 B. Antibody HIV Test
 C. Combination HIV antigen/antibody test
 D. CD4 count
The answer is A: Nucleic Acid Test (NAT)

14. A patient, who is in the Chronic Stage of HIV, has a CD4 count ordered. What does this test
measure?
 A. Red blood cells
 B. B cells
 C. Cytotoxic T cells

 D. Helper T cells
The answer is D: Helper T cells

15. What is a normal CD4 count?


 A. 200-500 cells/mm3
 B. 1500-3500 cells/mm3

 C. 500-1500 cells/mm3
 D. <200 cells/mm3
The answer is C: 500-1500 cells/mm3

16. A 48-year-old patient is HIV positive. The patient has no signs and symptoms and has a CD4
count of 400 cells/mm3. In addition, no opportunistic infections or diseases are present. These
findings correlate with what stage of HIV?
 A. Acute

 B. Chronic
 C. AIDS
The answer is B: Chronic. These findings correlate with the Chronic Stage (also called the
Asymptomatic Stage) of HIV. Signs and symptoms may not be experienced, the viral load is
lower than the Acute Stage, but the virus is still replicating and destroying the cells. The patient
can still transmit the virus to others. In addition, the CD4 count should be more than 200
cells/mm3 to about 500 cells/mm3. In addition, no opportunistic infections or diseases should be
present.

17. Select the criteria below that is used to help diagnosed a patient with Acquired
Immunodeficiency Syndrome (AIDS):

 A. CD4 count <200 cells/mm3

 B. Presence of opportunistic infection

 C. CD4 count >1500 cells/mm3

 D. WBC 9500
 E. Absence of opportunistic infection
The answers are A and B. A patient is diagnosed with AIDS if: CD4 count drops to less than 200
cells/mm3 or an opportunistic infection is present

18. A patient with AIDS has dark purplish brown lesions on the mucus membranes of the mouth.
As the nurse you know these lesions correlate with what type of opportunistic disease?
 A. Epstein-Barr Virus
 B. Herpes Simplex Virus
 C. Cytomegalovirus

 D. Kaposi’s Sarcoma
The answer is D: Kaposi’s Sarcoma
19. What sign and symptoms in your patient with HIV indicates the disease is worsening and the
immune system is severely compromised?
 A. Open, oozing lesions around the mouth

 B. White hair like spots on the side of the tongue


 C. Cheesy white film on the tonsils and inside cheeks
 D. Vision changes
The answer is B. This is known as oral hairy leukoplakia. It occurs when the immune system is
extremely compromised like with HIV and the Epstein-Barr virus. It is a signal the HIV is
getting worse.

20. A patient with AIDS has developed CMV (cytomegalovirus). The nurse makes it PRIORITY
to educate the patient about which of the following regarding CMV?

 A. Avoiding exposure to cat feces.


 B. Drinking water from clean sources only.
 C. Scheduling an eye appointment.
 D. Isolating for 14 days to prevent transmission of CMV to others.
The answer is C. CMV is an opportunistic infection that can develop in patients with AIDS. This
virus can cause damage to the retina (hence retinitis) and lead to blindness. The nurse should
educate the patient about scheduling an eye exam to monitor for this condition.

21. Your patient is in the last stage of HIV. The patient CD4 count is 100 cells/mm3. Which of
the following FUNGAL infections if your patient at risk for? Select all that apply:
 A. Mycobacterium tuberculosis

 B. Candidiasis

 C. Coccidioidomycosis
 D. Toxoplasmosis
 E. Histoplasmosis
The answers are B, C, and E. Candidiasis, Coccidioidomycosis, Histoplasmosis are all FUNGAL
infections. Mycobacterium tuberculosis is a bacterial infection, and Toxoplasmosis is a protozoal
(parasitic type) infection.

22. Which type of opportunistic infection occurs from inhaling a parasitic organism that can be
found in cat and bird feces?
 A. Histoplasmosis
 B. Crytococcosis
 C. Mycobacterium Avium Complex

 D. Toxoplasmosis
The answer is D. Toxoplasmosis is a protozoal parasitic infection that is found in cat and bird
feces along with undercook meat, such as pork and red meat.

23. You’re providing education to a patient with AIDS on how to prevent opportunistic
infections. Which statement below requires the nurse to re-educate the patient about this topic?
 A. “I’m traveling to Puerto Rico next week and will be sure to pack bottled water.”

 B. “I’ve switched to buying raw organic milk.”


 C. “Last month I received the Pneumovax.”
 D. “My neighbor bought a cat last week.”
The answer is B. Patients with AIDS should AVOID raw, undercooked, and unpasteurized foods
and beverages. These items could contain organisms that could cause serious infections to a
person with a compromised immune system.

24. The nurse plays a vital role in screening patients for a possible HIV infection. What questions
below could the nurse ask to help identify a patient who is at risk for HIV? Select all that apply:
 A. “How often do you use alcohol or drugs?”
 B. “Have you recently experienced an abusive relationship?”

 C. “If you are sexually active, do you or your partner use protection?”
 D. “In the past month, have you felt sad or unable to get out of bed?”

 E. “Have you ever been treated for a sexually transmitted infection?”


 F. “Do you ever experience intrusive or unwanted thoughts?”
The answers are A, C, E. The nurse should screen patients for HIV. Questions about sexual
behavior (if the patient is sexually active, how often, treatment for STI, barrier devices used, and
number of partners), drug/alcohol usage, and if they’ve had a blood transfusion before 1985, can
all help the nurse identify patients who may need to undergo HIV testing.

25. TRUE OR FALSE: The Center for Disease Control and Prevention (CDC) recommends that
all people between the ages of 13-64 be tested at least once for HIV during a routine health visit,
regardless of risk factors.

 True
 False
The answer is TRUE.

26. A 25-year-old patient reports that they engage in high risk activities that could lead to an HIV
infection. The patient’s test results show the patient is HIV-negative. The nurse should provide
the patient with education about?
 A. PEP

 B. PrEP
 C. Opportunistic Infections
 D. Nucleic Acid Test (NAT)
The answer is B. The nurse should provide the patient with education about PrEP (Pre-Exposure
Prophylaxis). These medications are taken BEFORE a possible encounter with HIV, which helps
prevent a possible HIV infection. The patient must be HIV-negative before taking these
medications.

27. Which patient below is a candidate for PEP (Post-exposure Prophylaxis)?


 A. A 32-year-old patient who reports sharing IV drug injection devices with a

person who is HIV-positive 5 days ago.


 B. A 28-year-old patient who engages in high risk activities on a regular basis that
could lead to an HIV infection.
 C. A 55-year-old who is HIV-positive.
 D. A 30-year-old who was sexually assaulted two days ago.
The answer is D. PEP (post-exposure prophylaxis) is medication that is taken AFTER an
encounter with an HIV infected person. These medications can help prevent becoming infected
with HIV, if started within 72 HOURS of the exposure (option A is not a candidate). These
medications are NOT for routine usage but for emergencies (sexual assault, needle stick etc.). If
a patient is at high risk (as with the patient in option B), they should consider PrEP (pre-exposure
prophylaxis). PEP is taken for 28 days.

28. A 25-year-old female is about to deliver a baby. The patient is HIV-positive and has been
taking antiretroviral therapy during the pregnancy. What steps can be taken to help prevent
transmitting the virus to the baby after birth?

 A. Substitute formula for breastfeeding.


 B. Administer antiretroviral treatment to the newborn for 2 weeks after birth.
 C. Avoid kissing and hugging the newborn.
 D. Stop taking antiretroviral therapy for 2 months postpartum.
The answer is A. Breastmilk can transmit the HIV virus. The patient should avoid breastfeeding
but use formula instead. Option B is wrong because the newborn should receive antiretroviral
treatment for 4-6 weeks after birth (NOT 2 weeks). The mother can kiss and hug her newborn
(this does NOT transmit the virus), and the patient should not quit taking antiretroviral therapy
during the postpartum period.

29. Which statement below is not a true statement about Antiretroviral Treatment?
 A. “The patient starts out taking 3 medications from at least 2 drug classes.”
 B. “ART decreases the amount of virus in the blood within about 6 months.”

 C. “ART helps decrease the risk of developing an opportunistic infection.”


 D. “Antiretroviral medications are taken when signs and symptoms appear and then
tapered off.”
The answer is D: This option is false and all the others are true regarding ART. ART must be
taken EXACTLY as prescribed (everyday, at the same time, at the right dosage etc.). It is not
taken when signs and symptoms appear and tapered off. If ART is not taken as prescribed or
doses are missed, drug resistance can develop. In other words, the medications will stop being
effective against the HIV.

30. Which class of antiretroviral medications bind with the CD4 receptors and inhibit the HIV’s
glycoprotein from being able to activate and engage the co-receptors?

 A. Post-attachment Inhibitors
 B. Fusion Inhibitors
 C. Integrase Inhibitors
 D. Non-nucleoside Reverse Transcriptase Inhibitors
The answer is A: Post-attachment Inhibitors. Trogarzo (ibalizumab) is a type of post-attachment
inhibitor. It is given IV every 2 weeks.

31. Fill-in-the-blank: Chemokine Receptor Antagonists block the co-receptor __________ on the
cell so HIV cannot engage the receptor and enter the cell.
 A. CXCR4
 B. CD4

 C. CCR5
 D. GP120
The answer is C: CCR5

32. The patient is prescribed to take Enfuvirtide (Fuzeon). The nurse prepares to administer this
medication via?
 A. Intravenous route
 B. Oral route
 C. Topical Route

 D. Subcutaneous Route
The answer is D.

33. Which antiretroviral medication treatment stops the enzyme reverse transcriptase from
working by binding to it?

 A. Non-nucleoside Reverse Transcriptase Inhibitors (NNRTIs)


 B. Integrase Inhibitors
 C. Protease Inhibitors
 D. Fusion Inhibitors
The answer is A.

34. A patient with Stage 2 HIV (Chronic Stage) is prescribed to take Raltegravir. The nurse
knows that this medication is part of what class of ART?
 A. Protease Inhibitors

 B. Integrase Inhibitors
 C. Nucleoside Reverse Transcriptase Inhibitors
 D. Attachment Inhibitors
The answer is B. Raltegravir is an Integrase Inhibitors. Remember from the lecture: “tegra” in
Integrase….the meds in this class have “tegra” in the middle…like RalTEGRAvir.

35. What group of ART for the treatment of HIV stop an enzyme from cutting the long chains of
virus so the immature virus can’t be assembled and mature?

 A. Protease Inhibitors
 B. Integrase Inhibitors
 C. Nucleoside Reverse Transcriptase Inhibitors
 D. Attachment Inhibitors
The answer is A.

36. A patient with HIV is prescribed to start antiretroviral therapy. The nurse is providing
education about these medications. Which statement below by the patient indicates they need re-
education on these medications?
 A. “If I take these medications as prescribed my viral load will become
undetectable, and I have a low risk of transmitting the virus to others.”
 B. “Drug resistance is likely to develop if I’m non-compliant with my
medications.”

 C. “I currently take a medication called St. John’s Wort to treat depression.”


 D. “This therapy does not cure me from HIV but helps me live a healthier and
longer life.”
The answer is C. Patients who take ART should be educated about how these medications can
interact with over-the-counter medications, especially herbal supplements like St. John’s Wort.
This medication is used to treat depression. The patient should be re-educated about this topic.
Cholecystitis*
1. The gallbladder is found on the __________ side of the body and is located under the
____________. It stores __________.
 A. right; pancreas; bilirubin
 B. left; liver; bile
 C. right; thymus' bilirubin

 D. right; liver; bile


The answer is D. The gallbladder is found in the RIGHT side of the body and is located under
the LIVER. It stores BILE.

2. Which statements below are CORRECT regarding the role of bile? Select all that apply:
 A. Bile is created and stored in the gallbladder.

 B. Bile aids in digestion of fat soluble vitamins, such as A, D, E, and K.

 C. Bile is released from the gallbladder into the duodenum.

 D. Bile contains bilirubin.


The answer are B, C, and D. Option A is INCORRECT because bile is created in the LIVER (not
gallbladder), but bile is stored in the gallbladder.

3. You’re providing a community in-service about gastrointestinal disorders. During your


teaching about cholecystitis, you discuss how cholelithiasis can lead to this condition. What are
the risk factors for cholelithiasis that you will include in your teaching to the participants? Select
all that apply:
 A. Being male
 B. Underweight

 C. Being female
 D. Older age

 E. Native American
 F. Caucasian

 G. Pregnant

 H. Family History

 I. Obesity
The answers are C, D, E, G, H and I. Cholelithiasis is the formation of gallstones. Risk factors
include: being female, older age (over 40), Native American or Mexican American descent,
pregnant, obesity, and family history.

4. A patient is being transferred to your unit with acute cholecystitis. In report the transferring
nurse tells you that the patient has a positive Murphy’s Sign. You know that this means:
 A. The patient stops breathing in when the examiner palpates under the ribs on the

right upper side of the abdomen at the midclavicular line.


 B. The patient stops breathing out when the examiner palpates under the ribs on the
right upper side of the abdomen at the midclavicular line.
 C. The patient verbalizes pain when the lower right quadrant is palpated.
 D. The patient reports pain when pressure is applied to the right lower quadrant but
then reports an increase in pain intensity when the pressure is released.
The answer is A. Murphy’s Sign can occur with cholecystitis. This occurs when the patient is
placed in the supine position and the examiner palpates under the ribs on the right upper side of
the abdomen. The examiner will have the patient breathe out and then take a deep breath in. The
examiner will simultaneously (while the patient is breathing in) palpate on this area under the
ribs at the midclavicular line (hence the location of the gallbladder). It is a POSITIVE Murphy’s
Sign when the patient stops breathing in during palpation due to pain.
5. Your patient is post-op day 3 from a cholecystectomy due to cholecystitis and has a T-Tube.
Which finding during your assessment of the T-Tube requires immediate nursing intervention?
 A. The drainage from the T-Tube is yellowish/green in color.

 B. There is approximately 750 cc of drainage within the past 24 hours.


 C. The drainage bag and tubing is at the patient's waist.
 D. The patient is in the Semi-Fowler's position.
The answer is B. A T-Tube should not drain more than about 500 cc of drainage per day (within
24 hours). A T-Tube’s drainage will go from bloody tinged (fresh post-op) to yellowish/green
within 2-3 days. The drainage bag and tubing should be below the site of insertion (at or below
the patient's waist so gravity can help drainage the bile), and the patient should be in Semi-
Fowler's to Fowler's position to help with draining the bile.

6. The physician orders a patient’s T-Tube to be clamped 1 hour before and 1 hour after meals.
You clamp the T-Tube as prescribed. While the tube is clamped which finding requires you to
notify the physician?
 A. The T-Tube is not draining.
 B. The T-Tube tubing is below the patient’s waist.

 C. The patient reports nausea and abdominal pain.


 D. The patient’s stool is brown and formed.
The answer is C. A nurse should ONLY clamp a T-Tube with a physician’s order. Most
physicians will prescribe to clamp the T-tube 1 hour before and 1 hour after meals. WHY? So,
bile will flow down into the small intestine (instead out of the body) during times when food is in
the small intestine to help with the digestion of fats. This is to help the small intestine adjust to
the flow of bile in preparation for the removal of the t-tube (remember normally it received bile
when the gallbladder contracted but now it will flow from the liver to the small intestine
continuously). Option C is an abnormal finding. The patient should not report nausea or
abdominal pain when the tube is blocked. This could indicate a serious problem. Option A is
correct because the T-tube should not be draining because it’s clamped. Option B is correct
because the T-tube tubing should be below or at the patient’s waist level. Option D is correct
because this shows the body is digesting fats and bilirubin is exiting the body through the stool
(remember bilirubin is found in the bile and gives stool its brown color…it would be light
colored if the bilirubin was not present). You would NOT want to see steatorrhea (fat/greasy
liquid stools) because this shows the bile isn’t being delivered to help digest the fats.

7. Your recent admission has acute cholecystitis. The patient is awaiting a cholecystostomy. What
signs and symptoms are associated with this condition? Select all that apply:
 A. Right lower quadrant pain with rebound tenderness
 B. Negative Murphy’s Sign

 C. Epigastric pain that radiates to the right scapula

 D. Pain and fullness that increases after a greasy or spicy meal

 E. Fever

 F. Tachycardia

 G. Nausea
The answers are C, D, E, F, and G. Option A and B are not associated with cholecystitis, but a
POSITIVE Murphy’s Sign is.

8. A patient in the emergency room has signs and symptoms associated with cholecystitis. What
testing do you anticipate the physician will order to help diagnose cholecystitis? Select all that
apply:
 A. Lower GI series

 B. Abdominal ultrasound

 C. HIDA Scan (Hepatobiliary Iminodiacetic AciD scan)


 D. Colonoscopy
The answers are B and C. These two tests can assess for cholecystitis. A lower GI series would
not assess the gallbladder but the lower portions of the GI system like the rectum and large
intestine. Option D is wrong because it would also assess the lower portions of the GI system.

9. You’re precepting a nursing student who is helping you provide T-Tube drain care. You
explain to the nursing student that the t-shaped part of the drain is located in what part of the
biliary tract?
 A. Cystic duct
 B. Common hepatic duct

 C. Common bile duct


 D. Pancreatic duct
The answer is C. The “T-shaped” part of the drain is located in the common bile duct and helps
deliver bile to the duodenum (small intestine).

10. Your patient is unable to have a cholecystectomy for the treatment of cholecystitis. Therefore,
a cholecystostomy tube is placed to help treat the condition. Which statement about a
cholecystostomy (C-Tube) is TRUE?
 A. The C-Tube is placed in the cystic duct of the gallbladder and helps drain

infected bile from the gallbladder.


 B. Gallstones regularly drain out of the C-Tube, therefore, the nurse should flush
the tube regularly to ensure patency.
 C. The C-Tube is placed through the abdominal wall and directly into the
gallbladder where it will drain infected bile from the gallbladder.
 D. The tubing and drainage bag of the C-Tube should always be level with the
insertion site to ensure the tube is draining properly.
The answer is C. This is the only correct statement about a cholecystostomy. A cholecystostomy,
also sometimes called a C-Tube, is placed when a patient can't immediately have the gallbladder
removed (cholecystectomy) due to cholecystitis. It is placed through the abdominal wall and into
the gallbladder. It will drain infected bile (NOT gallstones). The tubing and drainage bag should
be at or below waist level so it drains properly.

11. A patient, who has recovered from cholecystitis, is being discharged home. What meal
options below are best for this patient?

 A. Baked chicken with steamed carrots and rice


 B. Broccoli and cheese casserole with gravy and mashed potatoes
 C. Cheeseburger with fries
 D. Fried chicken with a baked potato
The answer is A. The patient should eat a low-fat diet and avoid greasy/fatty/gassy foods. Option
B is wrong because this contains dairy/animal fat like the cheese and gravy, and broccoli is
known to cause gas. Option C and D are greasy food options.

12. Your patient is diagnosed with acute cholecystitis. The patient is extremely nauseous. A
nasogastric tube is inserted with GI decompression. The patient reports a pain rating of 9 on 1-10
scale and states the pain radiates to the shoulder blade. Select all the appropriate nursing
interventions for the patient:
 A. Encourage the patient to consume clear liquids.

 B. Administered IV fluids per MD order.

 C. Provide mouth care routinely.

 D. Keep the patient NPO.

 E. Administer analgesic as ordered.

 F. Maintain low intermittent suction to NG tube.


The answers are B, C, D, E, and F. The treatment for cholecystitis includes managing pain,
managing nausea/vomiting (a NG tube with GI decompression (removal of stomach contents) to
low intermittent suction may be ordered to help severe cases), keep patient NPO until signs and
symptoms subside, mouth care from vomiting and nasogastric tube, and administer IV fluids to
keep the patient hydrated.

Crohn’s Disease
1. True or False: A patient with Crohn's Disease can experience inflammation in the large
intestine that affects mainly the mucosa (inner layer) of the bowel.
 True

 False
The answer is FALSE: A patient with Crohn's Disease can experience inflammation throughout
the GI Tract (mainly in the terminal ileum and beginning of the colon) from the mouth to anus
(not just the large intestine) and it affects the WHOLE bowel lining (not just the mucosa layer).

2. A patient with Crohn's Disease is MOST likely to have the disease is what part of the GI tract?
 A. Rectum
 B. Duodenum of the small intestine

 C. Terminal Ileum
 D. Descending colon
The answer is C. Crohn's disease is MOST likely to affect the terminal ileum. However, it may
affect any area of the GI tract.

3. You're providing teaching to a patient who has been newly diagnosed with Crohn's Disease.
Which statement by the patient's spouse requires re-education?
 A. "Crohn's Disease can be scattered throughout the GI tract in patches with some
areas appearing healthy while others are diseased."
 B. "There is no cure for Crohn's Disease."
 C. "Strictures are a common complication with Crohn's Disease."
 D. "Crohn's Disease can cause the haustra of the large intestine to lose its

form."
The answer is D. All the statements are true except option D. ULCERATIVE COLITIS can cause
the haustra of the large intestine to lose its form. This is not common with Crohn's Disease.

4. A physician is explaining to a patient that the patient has a type of Crohn's Disease that is
found in both the ileum and colon. As the nurse, you know this type of Crohn's Disease is called?
 A. Gastroduodenal Crohn's Disease
 B. Granulomatous Colitis
 C. Ileitis

 D. Ileocolitis
The answer is D. This patient has ileocolitis which affects parts of the colon and ileum.
Gastroduodenal Crohns affects the stomach and duodenum which is the first part of the small
intestine. Granulomatous colitis affects only the colon. Ileitis affects the ileum.

5. Select ALL of the following that are complications associated with Crohn's Disease:

 A. Cobble-stone appearance of GI lining


 B. Lead-pipe sign
 C. Toxic megacolon

 D. Fistula

 E. Abscess

 F. Anal Fissure
The answers are A, D, E, and F. These are all complications found with Crohn's Disease. Lead-
pipe sign and toxic megacolon are complications associated with ulcerative colitis.
6. Your patient with Crohn's Disease is admitted with an opening that has formed between the
bowel and bladder. As the nurse, you know this is what type of complication associated with this
disease?

 A. Enterovesical Fistula
 B. Rectovaginal Stricture
 C. Enteroenteric Fistula
 D. Perianal Fissure
The answer is A. These scenario describes a fistula which is an abnormal passage that forms deep
in the wall of the intestine to form an opening between intestine to intestine, intestine to organ, or
intestine to skin’s surface. This specific patient is experiencing an enterovesical fistula which is
an abnormal passage between the bowel and bladder.

7. A patient experiencing a flare-up with Crohn's Disease is ordered complete bowel rest by the
physician. You are administering TPN (total parental nutrition) per physician order. When
developing the patient's nursing plan of care, which nursing diagnosis is MOST important to
include in the care plan?
 A. Risk for allergy response
 B. Risk for unstable blood glucose level
 C. Risk for imbalance nutrition: more than body requirements

 D. Risk for imbalanced nutrition: less than body requirements


The answer is D. Patients with Crohn's Disease are at risk for undernourishment due to how the
disease process effects the body (small intestine is inflamed which is the area of the gut that
absorbs most of the nutrients from food). With severe cases of Crohn's Disease, the physician
may order complete bowel rest (where the patient will be nothing by mouth (NPO) and nothing
will enter the GI system....so the patient is at even more risk for imbalance nutrition. Physicians
may prescribe an IV solution (total parental nutrition) which will be given through a central line
that contains nutrients (so it enters the bloodstream...bypassing the gut). However, there is still a
risk for imbalanced nutrition (less than body requirements) for the patient. The nurse must
monitor the patient's nutrition status very closely such as daily weights, hydration status,
electrolytes, skin, etc.

8. A patient is receiving treatment for Crohn's Disease. Which food found on the patient's food
tray should the patient avoid?

 A. Fresh Salad
 B. White rice
 C. Baked chicken
 D. Cooked skinless apples
The answer is A. Patients who are experiencing flare-ups of Crohn's Disease should avoid high
fiber foods, foods that are hard to digest, spicy foods, dairy products etc. Therefore, the patient
should avoid a fresh salad. This contain vegetables which are high in fiber and hard to digest.
The gut needs to rest. It is best for the patient to consume low fiber and high protein foods. White
rice and fruits/vegetables that are cooked/skinless are low in fiber. Baked chicken is a good
source of protein for the patient.

9. A physician has prescribed a patient with a severe case of Crohn's Disease to take a drug that
works by suppressing the immune system. This medication achieves this by blocking a protein
that plays a role the inflammatory process. Which drug does this describe?
 A. Azathioprine
 B. Sulfasalazine

 C. Infliximab
 D. Prednisone
The answer is C. Infliximab (Remicade) is a TNF-blocker (biologic drug) which blocks tumor
necrosis factor which plays a role in the inflammatory response system. Azathioprine is an
immunosupressor which suppresses the immune system but does NOT block TNF. Sulfasalazine
is an 5-Aminosalicylate which is an anti-inflammatory medication. Prednisone is a corticosteroid
which decreases inflammation.
10. A patient with Crohn's Disease is taking corticosteroids. The patient is complaining of
extreme thirst, polyuria, and blurred vision. What is your next nursing action?

 A. Check the patient's blood glucose


 B. Give the patient a food containing sugar (ex: orange juice)
 C. Administer oxygen via nasal cannula
 D. D. Assess bowel sounds
The answer is A. A side effect of corticosteroids is hyperglycemia. Extreme thirst, polyuria, and
blurred vision are classic signs and symptoms of hyperglycemia. Therefore, the nurse should
check the patient's blood glucose to confirm the hyperglycemia.

Sexually Transmitted Disease

1. A patient has been diagnosed with primary syphilis (Stage I). When assessing the patient,
which of these findings will the healthcare provider anticipate? Choose 1 answer:
A. Reddish rash on the palms of the hands
B. Firm and painless genital ulcers
C. Sore throat and swollen lymph glands
D. Muscle weakness and visual changes

ANS; B

2. When teaching a group of students about chlamydia, which of the following points
is most important for the healthcare provider to emphasize?
A. Most people infected with chlamydia are unaware that they are infected
B. Burning and pain with urination is a frequent symptom of chlamydial infections
C. Chlamydia is the least common of all the major sexually transmitted diseases
D. Good handwashing technique is the best way to prevent chlamydial infections

ANS; A
3. The healthcare provider is teaching a youth group about how to avoid acquiring a sexually
transmitted disease (STD) if they are sexually active. Which of the following is the best method
of prevention?
A. Consistent condom use
B. Human papillomavirus (HPV) vaccination
C. Use of spermicidal creams
D. Douching after intercourse

ANS; A

4. A male patient arrives at the urgent care clinic reporting the following: painful and swollen
testicles, burning with urination, and a yellow-green discharge from the penis. The healthcare
provider understands that these symptoms are likely due to which of these problems?
A. Stage 2 syphilis
B. Epididymitis
C. Gonorrhea
D. Trichomoniasis

ANS; C

5. A patient diagnosed with trichomoniasis is being treated with metronidazole (Flagyl). When
teaching the patient about this medication, which of the following will the healthcare provider
include?
A. “Call our office if you experience any tendon pain or tenderness.”
B. “Report the occurrence of pain in your upper abdomen immediately.”
C. “You should avoid milk or dairy products during therapy.”
D. “Do not drink alcohol while you are taking this medication.”

ANS; D
6. A patient asks the healthcare provider about the benefits of receiving the human
papillomavirus (HPV) vaccine. Which statement is the most appropriate response by the
healthcare provider?
A. “You will need to have a booster vaccination each year.”
B. “You will no longer need to get a routine cervical exam.”
C. “The HPV vaccine will protect you from all types of the virus.”
D. The HPV vaccine can help prevent cervical cancer.”

ANS; D

7. The healthcare provider is assessing a female patient diagnosed with trichomoniasis. Which of
the following characteristics would the healthcare provider use to describe the vaginal discharge
of this patient?
A. White and curd-like
B. Frothy and malodorous
C. Clear and watery
D. Scant and blood-tinged

ANS; B

8. A patient presents to the clinic with a report of joint pain, malaise, fever, and a rash on the
palms of the hands. Based on these findings, which action should the healthcare provider
implement next?
A. Inquire about the patient’s recent sexual activity
B. Draw blood for laboratory analysis
C. Inquire about the development of genital lesions
D. Administer a fever-reducing medication

ANS; C
9. A woman is admitted to the medical unit with a report of sudden, severe, sharp, pain in the
lower abdomen and an absence of menses of 8 weeks. When interviewing the patient, about
which of the following will the healthcare provider inquire?
A. History of chlamydial infection
B. Frequency of sexual activity
C. Sexual orientation
D. Intravenous drug use

ANS; A

10. During her first prenatal visit, a woman asks why she is being screened for syphilis. Which of
the following is the best response by the healthcare provider?
A. “If you have the infection, the safest time to treat you is during the first trimester.”
B. “Syphilis can be transferred from you to your baby through the placenta.”
C. "We must report all cases of syphilis to the health department.”
D. “If you test positive you may need to be admitted to the hospital for care.”

ANS; B

PERITONITIS
1. During the assessment, the nurse should be looking for additional symptoms diagnostic of
peritonitis, which include:
A. Abdominal rigidity.
B. Diminished peristalsis.
C. Leukocytosis.
D. All of the above.

ANS; D

2. The nurse should continue to assess for the common complications of:
A. Abscess formation.
B. Respiratory arrest.
C. Umbilical hernia.
D. Urinary tract infection.

ANS; A
3. The major cause of death from peritonitis is:
A. Hypovolemia.
B. Sepsis.
C. Shock.
D. Abscess formation.

ANS; B

4. The following are results of diagnostic and imaging studies in a patient with peritonitis except:
A. Elevated WBC.
B. Air and fluid levels on abdominal xray.
C. Leukopenia.
D. Intra-abdominal abscess on MRI.

ANS; C

5. Peritonitis occurs in which part of the GI system?


A. Peritoneum.
B. Appendix.
C. Duodenum.
D. Sigmoid colon

ANS; A

6. Abdominal rigidity and decreased bowel sounds would raise your suspicions of what
condition?
A. Cholecystitis
B. Peritonitis
C. Cystitis
D. Ulcerative colitis

ANS; B

PANCREATITIS
1. Inside the pancreas are special cells that secrete digestive enzymes and hormones. The cells
that secrete digestive enzymes are known as ______________ cells.
A. Islet of Langerhans
B. Protease
C. Acinar
D. Amylase
ANS;C
2. From the pancreas and gallbladder, the common bile duct and pancreatic duct open into the
____________ where digestive enzymes and bile flow into the duodenum via the major
duodenal papilla which is surrounded by a muscular valve that controls the release of digestive
enzymes known as the ______________.
A. ampulla of vater, sphincter of Oddi
B. papilla of vater, sphincter of Oddi
C. minor duodenal papilla, ampulla of vater
D. jejunum, sphincter of pylori
ANS; A

3. Select-ALL-that-apply: In the pancreas, the acinar cells release:


A. Amylase
B. Somatostatin
C. Lipase
D. Protease
ANS; A,C, D,
4. You're caring for a 45 year old patient who is admitted with suspected acute pancreatitis. The
patient reports having extreme mid-epigastric pain that radiates to the back. The patient states the
pain started last night after eating fast food. As the nurse, you know the two most common
causes of acute pancreatitis are:
A. High cholesterol and alcohol abuse
B. History of diabetes and smoking
C. Pancreatic cancer and obesity
D. Gallstones and alcohol abuse
ANS; D

5. Which patient below is at MOST risk for CHRONIC pancreatitis?


A. A 25-year-old female with a family history of gallstones.
B. A 35-year-old male who reports social drinking of alcohol.
C. A 15-year-old female with cystic fibrosis
D. A 66-year-old female with stomach cancer
ANS; C

6. Your patient with acute pancreatitis is scheduled for a test that will use a scope to assess the
pancreas, bile ducts, and gallbladder. The patient asks you, "What is the name of the test I’m
going for later today?" You tell the patient it is called:
A. MRCP
B. ERCP
C. CT scan of the abdomen
D. EGD
ANS; B

7. A patient is admitted to the ER with the following signs and symptoms: very painful mid-
epigastric pain felt in the back, elevated glucose, fever, and vomiting. During the head-to-toe
assessment, you notice bluish discoloration around the belly button. As the nurse, you know this
is called?
A. Grey-Turner's Sign
B. McBurney’s Sign
C. Homan’s Sign
D. Cullen’s Sign
ANS; D

8. While assisting a patient with chronic pancreatitis to the bathroom, you note the patient's stool
to be oily/greasy in appearance. In your documentation you note this as:
A. Steatorrhea
B. Melena
C. Currant
D. Hematochezia
ANS; A
9. A patient with acute pancreatitis is reporting excessive thirst, excessive voiding, and blurred
vision. As the nurse, it is priority you?
A. Reassure the patient this is normal with pancreatitis
B. Check the patient's blood glucose
C. Assist the patient with drinking a simple sugar drink like orange juice
D. Provide a dark and calm environment
ANS; B

10. A patient who received treatment for pancreatitis is being discharged home. You're providing
diet teaching to the patient. Which statement by the patient requires immediate re-education
about the diet restrictions?
A. "It will be hard but I will eat a diet low in fat and avoid greasy foods."
B. "It is very important I limit my alcohol intake to no more than 2-3 glasses of wine a
week."
C. "I will concentrate on eating complex carbohydrates rather than refined
carbohydrates."
D. "I will purchase foods that are high in protein."
ANS; B

11. The physician orders a patient with pancreatitis to take a pancreatic enzyme. What
assessment finding demonstrates the pancreatic enzymes are working properly?
A. Abdominal girth is decreased
B. Skin turgor is less than 2 seconds
C. Blood glucose is 250
D. Stools appear formed and solid
ANS; D

12. During a home health visit, you are assessing how a patient takes the prescribed pancreatic
enzyme. The patient is unable to swallow the capsule whole, so they open the capsule and mix
the beads inside the capsule with food/drink. Which food or drink is safe for the patient to mix
the beads with?
A. Pudding
B. Ice cream
C. Milk
D. Applesauce
ANS; D

Crohn’s Disease vs Ulcerative Colitis [SAS 27]

1. True or False: Crohn’s Disease and ulcerative colitis are two forms of IBS (irritable bowel
syndrome).

Ans; false

2. Select ALL the options below that are similarities between ulcerative colitis and Crohn’s
Disease:
A. Each cause inflammation
B. Both affect the serosa layer
C. Each are found from the mouth to anus
D. They both increase colon cancer risk
E. The cause of both is unknown
F. The cure for both diseases includes total colectomy

Ans; a,d,e

3. Which medications are used in the treatment of Crohn’s Disease and ulcerative colitis?
SELECT-ALL-THAT-APPLY:
A. Guanylate Cyclase-C agonists
B. Anticholinergics
C. 5-Aminosalicylates
D. Antacids
E. Corticosteroids
F. Immune suppressors

ANS; C,E ,F

4. ______________ is most commonly found in the terminal ileum and beginning of the colon.
A. Ulcerative Colitis
B. Crohn’s Disease

ANS ; B

5. ______________ affects the inner layer of the intestinal lining.


A. Ulcerative Colitis
B.Crohn’s Disease
ANS; A

6. Which bowel disease starts in the rectum and migrates in a continuous fashion throughout the
colon?
A. Ulcerative Colitis
B. Crohn’s Disease

ANS; A.

7. Which of the following is NOT a potential complication associated with ulcerative colitis?
A. Toxic megacolon
B. Anemia
C. Stricture
D. Peritonitis

ANS; C

8. A patient has the following symptoms: urgent and frequent bowel movements of diarrhea that
contains blood with pus and mucous, low hemoglobin/hematocrit, potassium level of 2.0. Based
on the patient’s signs and symptoms, which disease does this describe?
A. Ulcerative Colitis
B.Crohn’s Disease

ANS; A

9. A patient had a colonoscopy which showed a “cobble-stone” appearance of the GI lining. This
is found in:
A. Ulcerative Colitis
B. Crohn’s Disease

ANS; B

10. A patient with severe ulcerative colitis had a barium enema. Which finding is associated with
this disease?
A. Cobble-stone appearance of the colon
B. Cullen’s Sign
C. Lead-Pipe Sign
D. McBurney’s Sign

ANS; C

11. Which type of bowel disease is most likely to cause severe malnourishment?
A. Ulcerative Colitis
B. Crohn’s Disease
ANS; B

12. Select-ALL-the complications associated with Crohn’s Disease:


A. Loss of form to the haustra
B. Fistulas
C. Strictures
D. Hemorrhoids
E. Anal Fissure

ANS; B,C,E

13. A patient has the following signs and symptoms: abdominal cramping which is mainly
located in the right lower side, ulcers in the mouth, bleeding anal fissure, and diarrhea. Based on
the patient’s signs and symptoms, which disease does this describe?
A. Ulcerative Colitis
B. Crohn’s Disease

ANS; B

Cystitis (Urinary Tract Infection)


1. You’re assessing your patients during morning rounding. Which patient below is at MOST
risk for developing a urinary tract infection?
A. A 25 year old patient who finished a regime of antibiotics for strep throat 10 weeks
ago.
B. A 55 year old female who is post-opt day 7 from hip surgery.
C. A 68 year old male who is experiencing nausea and vomiting.
D. A 87 year old female with Alzheimer’s disease who is experiencing bowel
incontinence.

ANS; D

2. A 36 year old female, who is 29 weeks pregnant, reports she is experiencing burning when
voiding. The physician orders a urinalysis. Which statement by the patient demonstrates she
understands how to 1collect the specimen?
A. “I’ll hold the cup firmly against the urethra while collecting the sample.”
B. “I will cleanse back to front with the antiseptic wipe before peeing in the cup.”
C. “First, I will pee a small amount of urine in the toilet and then collect the rest in the
cup.”
D. “I will be sure to drink a lot of fluids to keep the urine diluted before peeing into the
cup.”

ANS; C

3. During a head-to-toe assessment on a patient with a possible urinary tract infection, you
perform
costovertebral angle percussion. The costovertebral angle is found?
A. between the bottom of the 12th rib and spine
B. between the right upper quadrant and umbilicus
C. between the sternal notch and angle of Louis
D. between the ischial spine and umbilicus

ANS; A.

4. A 76-year-old female is admitted due to a recent fall. The patient is confused and agitated. The
family members report that this is not normal behavior for the patient. They explain that the
patient is very active in the community and cares for herself. Based on the information you have
gathered about the patient, which physician’s order takes priority?
A. “Collect a urinalysis”
B. “Collect a T3 and T4 level”
C. “Insert a Foley Catheter”
D. “Keep patient NPO”

ANS; A

5. The physician orders a urine culture on your patient in room 5505 with a urinary tract
infection. In addition, the patient is ordered to start IV Bactrim
(Sulfamethoxazole/Trimethoprim). How will you proceed with following this order?
A. First, hang the antibiotic, and then collect the urine culture.
B. First, hang the antibiotic and when the antibiotic is finished infusing collect the urine
culture.
C. First, collect the urine culture, and then hang the antibiotic.
D. First, collect the urine culture and then hold the dose of the antibiotic until the urine
culture is back from the lab.

ANS; C.

6. A patient with a urinary tract infection is taking Bactrim (Sulfamethoxazole/Trimethoprim).


As the nurse you know it is important that the patient consumes 2.5 to 3 L of fluid per day to
prevent which of the following complications?
A. Brown urine
B. Crystalluria
C. Renal Stenosis
D. Renal Calculi

ANS; B.

7. You’re providing discharge teaching to a female patient on how to prevent urinary tract
infections. Which statement is INCORRECT?
A. “Void immediately after sexual intercourse.”
B. “Avoid wearing tight fitting underwear.”
C. “Try to void every 2-3 hours”
D. “Use scented sanitary napkins or tampons during menstruation.”

ANS;D.

8. A patient, who is having spasms and burning while urinating due to a UTI, is prescribed
“Pyridium” (Phenazopyridine). Which option below is a normal side effect of this drug?
A. Hematuria
B. Crystalluria
C. Urethra mucous
D. Orange colored urine

ANS; D

9. You’re caring for a patient with an indwelling catheter. The patient complains of spasm like
pain at the catheter insertion site. Which of the following options below are other signs and
symptoms the patient could experience or the nurse could observe if a urinary tract infection was
present? SELECT-ALL-THAT-APPLY:
A. Increased WBC
B. Crystalluria
C. Positive McBurney’s Sign
D. Feeling the need to void even though a catheter is present
E. Dark and cloudy urine
F. Cramping

ANS; A,D,E,F,

10. On your nursing care plan for a patient with a urinary tract infection, which of the following
would be appropriate nursing interventions? SELECT-ALL-THAT-APPLY:
A. Encourage voiding every 2-3 hours while awake.
B. Restrict fluid intake to 1-2 liters per day.
C. Monitor intake and output daily.
D. The patient verbalizes the importance of using vaginal sprays to decrease reoccurrence
of
urinary tract infections prior to discharge home.

ANS;A , C

Urolithiasis (Kidney stones/ Renal Calculi)

1. You’re providing an in-service to a group of nurses about the different types of kidney stones.
You explain to the attendees that the most common type of kidney stone is made up of:
A. Cholesterol
B. Calcium and oxalate
C. Calcium and phosphate
D. Uric acid
ANS; B

2. Which patient below is at MOST risk for developing uric acid type kidney stones?
A. A 53 year old female with recurrent urinary tract infections.
B. A 6 year old male with cystinuria.
C. A 63 year male with gout.
D. A 25 year old female that follows a vegan diet and report eating high amounts of
spinach and strawberries on a regular basis.

ANS; C

3. A patient is scheduled for an intravenous pyelogram (IVP) to assess for kidneys stones. Which
finding below requires the nurse to contact the physician?
A. Patient reports flank pain that radiates downward
B. Patient has hematuria
C. Patient is allergic to shellfish
D. Patient has cloudy urine

ANS; C

4. The physician orders a 24-hour urine collection on a patient with recurrent kidney stones. As
the nurse you know that the specimen should be?
A. Kept at room temperature
B. Kept on ice or refrigerated
C. Sent to the lab every four hours
D. Kept at a temperature between 98.6 ‘F to 99.3’F

ANS; B

5. You’re providing care to a patient with a uric acid kidney stone that is 2 mm in size per
diagnostic imaging. The patient is having severe pain and rates their pain 10 on 1-10 scale. The
physician has ordered a treatment plan to assist the patient in passing the kidney stone. What
nursing intervention is PRIORITY for this patient based on the scenario?
A. Administer pain medication
B. Encourage fluid intake of 2-4 liters per day
C. Massage the costovertebral area
D. Implement a high protein diet

ANS; A.

6. A patient with a kidney stone explains that the pain he is experiencing is intense, sharp, and
wavelike that radiates to the scrotum. In addition, he explains it feels like he has to void but a
small amount of urine is passed. Based on the patient’s signs and symptoms, where may the
kidney stone be located?
A. Renal Calyx
B. Renal Papilla
C. Ureter
D. Urethra

ANS; C

7. You’re developing a nursing care plan for a patient with a kidney stone. Which of the
following nursing interventions will you include in the patient’s plan of care?
A. Restrict calcium intake
B. Strain urine with every void
C. Keep patient in supine position to alleviate pain
D. Maintain fluid restriction of 1-2 Liter per day

ANS; B

8. You’re providing discharge teaching to a patient who was hospitalized for the treatment of a
kidney stone. The type of kidney stone the patient experienced was a uric acid type stone. What
type of foods will you educate the patient to avoid?
A. Cabbage, spinach, tomatoes, strawberries
B. Ice cream, milk, pork, cheese
C. Beans, potatoes, corn, peas
D. Liver, scallops, anchovies, sardines, pork

ANS; D

9. Your patient arrives back to their room after having extracoporeal shock wave lithotripsy
(ESWL) for treatment of a kidney stone. What will be included in the patient’s plan of care?
SELECT-ALL-THAT- APPLY:
A. Keep the patient in bed
B. Encourage fluid intake of 3-4 liters per day
C. Maintain nephrostomy tube
D. Strain urine
E. Keep dressing dry and intact

ANS; B, D

10. You are providing pre-op teaching to a patient scheduled for a percutaneous
nephrolithotomy. Which statement by the patient demonstrates the patient understood the pre-
op teaching?
A. “During the procedure the surgeon will move the stone down the ureter, so I can pass
the stone in the urine.
B. “I may have a nephrostomy tube after the procedure.”
C. “A scope is inserted through the urinary system from the urethra to the kidneys to
assess the kidney stone.”
D. “This procedure is noninvasive and no incision is required.”
ANS; B

Pelvic Inflammatory Disease

1. A client comes to the clinic for treatment of recurrent pelvic inflammatory disease. The nurse
recognizes that this condition most frequently follows which type of infection?
A. Trichomoniasis
B. Chlamydia
C. Staphylococcus
D. Streptococcus

Correct Answer: B.
 Chlamydia
Explanation
 Chlamydial infections are one of the most frequent causes of salpingitis or pelvic
inflammatory disease.

2. What is pelvic inflammatory disease (PID)?


A. Abdominal distention
B Urinary tract infection
C. Infection of a woman's reproductive organs
D. Infertility

Correct answer : C
 Infection of a woman's reproductive organs
Explanation:
 Pelvic inflammatory disease (PID) is an infection of a woman's reproductive organs,
which includes the uterus, cervix, ovaries, and fallopian tubes.

PID is often caused by sexually transmitted infections (STIs, also referred to as sexually
transmitted diseases, or STDs), such as chlamydia and gonorrhea. However, PID may
also be cause by other infections that are not sexually transmitted.

3. What are risk factors for getting pelvic inflammatory disease?


A. Untreated STIs (STDs)
B. Multiple sexual partners
C. Douching
D. All of the above

Correct answer: D
 All of the above
Explanation:
 Risk factors for developing pelvic inflammatory disease include:
.. Untreated sexually transmitted infections (STIs)
.. Multiple sexual partners
.. Having a sexual partner who has multiple sexual partners besides you
.. Douching
.. Previous PID infection
.. Being sexually active and age 25 or younger
.. Using an intrauterine device (IUD) for birth control (this risk is mostly limited
to the first
three weeks after the IUD is inserted)

4. What are symptoms of pelvic inflammatory disease?


A. Abdominal pain
B. Foul-smelling vaginal discharge
C. Painful sexual intercourse
D. All of the above of the above

Correct answer: D
 All of the above
Explanation:
 Symptoms of pelvic inflammatory disease may range from mild to severe. Some women
do not experience any symptoms. When symptoms of PID occur, they may include
.. Pain in the lower abdomen (most common symptom)
.. Fever
.. Unusual vaginal discharge with a foul odor
.. Painful sexual intercourse and/or bleeding when you have sex
.. Bleeding between periods/irregular menstrual periods
.. Burning sensation or pain when urinating
.. Pain in the upper right abdomen (rare)
5. How is pelvic inflammatory disease diagnosed?
A. Pelvic exam
B. MRI
C. Endoscopy
D. X-ray
Correct answer is A:
 Pelvic exam
Explanation:
 There are no specific tests for pelvic inflammatory disease, but it can often be diagnosed
during a pelvic exam. A doctor will check for any pain or tenderness and abnormal
vaginal discharge. Swabs may be taken from the inside of the vagina and cervix to test
for chlamydia, gonorrhea, and sexually transmitted other infections. Samples of urine,
blood, and/or fluids from your vagina and cervix may also be taken.

Other tests that may be performed to confirm a diagnosis of PID include ultrasound,
endometrial biopsy (removal of a tissue sample from the lining of the uterus), and
laparoscopy (a tiny camera is inserted through a small cut in the belly button to visualize
the reproductive organs).

6. Pelvic inflammatory disease can increase the risk of developing some cancers.
A. True
B. False

Correct answer: A
 True
Explanation:
 Pelvic inflammatory disease may increase the risk of developing both ovarian and
cervical cancers. Many women with PID have HPV (human papilloma virus, the virus
that causes genital warts), which is a risk factor for cervical cancer. The increased risk of
ovarian cancer is small.

7. The chances of getting pregnant ____ if you have had pelvic inflammatory disease.
A. Increase
B. Decrease

Correct answer: B
 Decrease
Explanation:
 If you have had pelvic inflammatory disease more than once the chances of getting
pregnant may be lower. In PID, bacteria can enter the fallopian tubes and can result in
scarring, which can block an egg traveling from the ovary to the uterus.

8. What are complications of pelvic inflammatory disease?


A. Endometriosis
B. Uterine fibroids
C. Infertility
D. Incontinence

Correct answer : C
 Infertility
Explanation:
 If pelvic inflammatory disease is not diagnosed and treated early, complications may
include:
.. Scar tissue in the fallopian tubes that can lead to tubal blockage
.. Ectopic pregnancy (pregnancy outside the uterus)
.. Infertility
.. Chronic pelvic/abdominal pain

9. The risk of developing pelvic inflammatory disease can be reduced by…


A. Having only one sexual partner
B. Using condoms
C. Not douching
D. All of the above

Correct answer: D
 All of the above
Explanation:
 Pelvic inflammatory disease is not always preventable, since it may be caused by normal
bacteria in the vagina that can travel up to the reproductive organs. To reduce the risk of
developing PID, do not douche. This removes healthy bacteria that can protect against
infection.
 The other way to reduce the risk of developing PID is to prevent sexually transmitted
infections (STIs). The most effective way to do this is to avoid vaginal, oral, or anal sex
but this is not always possible or desirable so if you do have sex, take the following
precautions to prevent STI:
.. Use condoms. Other methods of birth control will not protect against STIs.
.. Limit your number of sexual partners.
.. Get tested for STIs before having sex, and remain monogamous after getting
tested.
.. Do not abuse alcohol or drugs, which can increase risky behavior and result in
unintended exposure to STIs.

10. Can pelvic inflammatory disease be cured?


A. Yes
B. NoNo

Correct answer: A
 Yes
Explanation:
 Antibiotics can cure pelvic inflammatory disease when it is diagnosed early. Take the
entire course of antibiotics as prescribed even if you're feeling better, to make sure the
infection goes away completely. In severe cases of PID, hospitalization with intravenous
antibiotics may be required. It is common for sexual partner(s) to also be treated as they
may still be infected with the bacteria that can cause PID.

Treatment won't undo any damage that has already occurred, such as scarring of the
fallopian tubes. Delayed treatment for PIDS increases the risk of complications such as
infertility or future ectopic pregnancy.

PID infection can recur if you are infected with another STI, and once you have had PID
your chances of developing it again are increased.

Benign Prostatic Hypertrophy (Hyperplasia) [BPH]

1. After undergoing a transurethral resection of the prostate to treat benign prostatic hypertrophy,
a patient is retuned to the room with continuous bladder irrigation in place. One day later, the
patient reports bladder pain. What should the nurse do first?
A. Increase the I.V. flow rate
B. Notify the doctor immediately
C. Assess the irrigation catheter for patency and drainage
D. Administer meperidine (Demerol) as prescribed
Correct answer: C.
 Assess the irrigation catheter for patency and drainage .
Explanation:
 Although postoperative pain is expected, the nurse should ensure that other factors, such
as an obstructed irrigation catheter, aren’t the cause of the pain. After assessing catheter
patency, the nurse should administer an analgesic such as meperidine as prescribed.
Increasing the I.V. flow rate may worse the pain. Notifying the doctor isn’t necessary
unless the pain is severe or unrelieved by the prescribed medication.

2. Alpha blockers in the treatment of BPH are associated with all of the following except:
A. Improvement in symptoms in 2-3 weeks
B. Side effects of dizziness and fatigue
C. Once daily dosing
D. 25% decrease in prostate size
E. Minimal changes in blood pressure

Correct answer: D.
 25% decrease in prostate size

3. Of the options below, is NOT a symptom of BPH?


A. Urinary incontinence
B. Difficulty urinating
C. Intestinal pain
D. Sexual dysfunction

Correct answer. C
 Intestinal pain.
Explanation:
 Recent studies suggest that there is a correlation between lower urinary tract symptoms
and sexual dysfunction in aging patients. In fact, the severity of urinary symptoms and
the degree of sexual dysfunction are strongly correlated, independently of age. In
particular, community-based studies have found that a significant number of patients with
symptomatic BPH have sexual dysfunction.

4. An eighty-five-year old man was admitted for surgery for benign prostatic hypertrophy.
Preoperatively he was alert, oriented, cooperative, and knowledgeable about his surgery. Several
hours after surgery, the evening nurse found him acutely confused, agitated, and trying to climb
over the protective side rails on his bed. The most appropriate nursing intervention that will
calm an agitated client is
A. limit visits by staff.
B. encourage family phone calls.
C. position in a bright, busy area.
D. speak soothingly and provide quiet music.

Correct answer: D.
 speak soothingly and provide quiet music.
Explanation:
 The client needs frequent visits by the staff to orient him and to assess his safety. Phone
calls from his family will not help a client who is trying to climb over the side rails and
may even add to his danger. Putting the client in a bright, busy area would probably add
to his confusion. The environment is an important factor in the prevention of injuries.
Talking softly and providing quiet music have a calming effect on the agitated client.

5.What is the relationship between prostate cancer and the condition of an enlarged prostate, also
known as BPH?
A. BPH and prostate cancer are unrelated
B. BPH causes prostate cancer
C. BPH is a symptom of prostate cancer

Correct answer: A.
 BPH and prostate cancer are unrelated

6. How common is BPH?


A. 10% of men in their 60s
B. 30% of men in their 60s
C. 50% of men in their 60s
D. 90% of men in their 60s

Correct answer: C.
 50% of men in their 60s.
Explanation:
 Although the prostate continues to grow during most of a man’s life, the enlargement
doesn’t usually cause problems until late in life. BPH rarely causes symptoms before the
age of 40, but more than half of men in their 60s, and as many as 90% in their 70s and
80s, have some symptoms of BPH

7. Romeo Diaz, age 78, is admitted to the hospital with the diagnosis of benign prostatic
hyperplasia (BPH). He is scheduled for a transurethral resection of the prostate (TURP). It
would be inappropriate to include which of the following points in the preoperative teaching?
A. TURP is the most common operation for BPH.
B. Explain the purpose and function of a two-way irrigation system.
C. Expect bloody urine, which will clear as healing takes place.
D. He will be pain free.

Correct answer: D.
 He will be pain free.
Explanation:
 Surgical interventions involve an experience of pain for the client which can come in
varying degrees. Telling the pain that he will be pain free is giving him false reassurance.

8. The obstructive and irritative symptom complex caused by benign prostatic hypertrophy is
termed
A. prostatism.
B. prostatitis.
C. prostaglandin.
D. prostatectomy.

Correct answer: A.
 prostatism.
Explanation:
 Symptoms of prostatism include increased frequency of urination, nocturia, urgency,
dribbling, and a sensation that the bladder has not completely emptied. Prostatitis is an
inflammation of the prostate gland.Prostaglandins are physiologically active substances
present in tissues with vasodilator properties. Prostatectomy refers to the surgical removal
of the prostate gland.

9. Transurethral resection of the prostate (TURP) is associated with all of the following except:
A. Outpatient procedure in most cases
B. Transfusion rate of 2-4%
C. Retrograde ejaculation in 50% of men
D. Most effective long term treatment
E. Erectile dysfunction in 2%

Correct answer: A.
 Outpatient procedure in most cases

10. A client is admitted to the hospital with benign prostatic hyperplasia, the nurse most relevant
assessment would be:
A. Flank pain radiating in the groin
B. Distention of the lower abdomen
C. Perineal edema
D. Urethral discharge

Correct answer: B.
 Distention of the lower abdomen.
Explanation:
 This indicates that the bladder is distended with urine, therefore palpable.

11. What question would be most important to ask a male client who is in for a digital rectal
examination?
A. “Have you noticed a change in the force of the urinary system?”
B. “Have you noticed a change in tolerance of certain foods in your diet?”
C. “Do you notice polyuria in the AM?”
D. “Do you notice any burning with urination or any odor to the urine?”

Correct answer: A.
 “Have you noticed a change in the force of the urinary system?”
Explanation:
 This change would be most indicative of a potential complication with (BPH) benign
prostate hypertrophy.

12. The nurse is administering a psychotropic drug to an elderly client who has history of benign
prostatic hypertrophy. It is most important for the nurse to teach this client to:
A. Add fiber to his diet.
B. Exercise on a regular basis.
C. Report incomplete bladder emptying
D. Take the prescribed dose at bedtime.

Correct answer: C.
 Report incomplete bladder emptying.
Explanation:
 Urinary retention is a common anticholinergic side effect of psychotic medications, and
the client with benign prostatic hypertrophy would have increased risk for this problem.
Adding fiber to one’s diet and exercising regularly are measures to counteract another
anticholinergic effect, constipation. Depending on the specific medication and how it is
prescribed, taking the medication at night may or may not be important. However, it
would have nothing to do with urinary retention in this client.

Heppa A, B, C, D, E
1. The liver receives blood from two sources. The _____________ is responsible for pumping
blood rich in nutrients to the liver.

A. hepatic artery

B. hepatic portal vein

C. mesenteric artery

D. hepatic iliac vein

The answer is B. The liver receives blood from two sources. The hepatic portal vein is
responsible for pumping blood rich in nutrients to the liver.

2. Which statements are INCORRECT regarding the anatomy and physiology of the liver?
Select all that apply:

A. The liver has 3 lobes and 8 segments.

B. The liver produces bile which is released into the small intestine to help digest fats.

C. The liver turns urea, a by-product of protein breakdown, into ammonia.

D. The liver plays an important role in the coagulation process.


The answers are A and C. The liver has 2 lobes (not 3), and the liver turns ammonia (NOT urea),
which is a by-product of protein breakdown, into ammonia. All the other statements are true
about liver’s anatomy and physiology.

3. You’re providing an in-service on viral hepatitis to a group of healthcare workers. You are
teaching them about the types of viral hepatitis that can turn into chronic infections. Which types
are known to cause ACUTE infections ONLY? Select all that apply:

A. Hepatitis A

B. Hepatitis B

C. Hepatitis C

D. Hepatitis D

E. Hepatitis E

The answers are A and E. Only Hepatitis A and E cause ACUTE infections…not chronic.
Hepatitis B, C, and D can cause both acute and chronic infections.

4. Which patients below are at risk for developing complications related to a chronic hepatitis
infection, such as cirrhosis, liver cancer, and liver failure? Select all that apply:

A. A 55-year-old male with Hepatitis A.

B. An infant who contracted Hepatitis B at birth.

C. A 32-year-old female with Hepatitis C who reports using IV drugs.

D. A 50-year-old male with alcoholism and Hepatitis D.

E. A 30-year-old who contracted Hepatitis E.

The answers are B, C, and D. Infants or young children who contract Hepatitis B are at a very
high risk of developing chronic Hepatitis B (which is why option B is correct). Option C is
correct because most cases of Hepatitis C turn into chronic cases and IV drug use increases this
risk even more. Option D is correct because Hepatitis D occurs when Hepatitis B is present and
constant usage of alcohol damages the liver. Therefore, the patient is at high risk of developing
chronic hepatitis. Hepatitis A and E tend to only cause acute infections….not chronic.

5. A patient is diagnosed with Hepatitis A. The patient asks how a person can become
infected with this condition. You know the most common route of transmission is?

A. Blood

B. Percutaneous
C. Mucosal

D. Fecal-oral

The answer is D. Hepatitis A is most commonly transmitted via the fecal-oral route.

6. Which of the following is NOT a common source of transmission for Hepatitis A? Select
all that apply:

A. Water

B. Food

C. Semen

D. Blood

The answers are C and D. The most common source for transmission of Hepatitis A is water and
food.

7. A 36-year-old patient’s lab work show anti-HAV and IgG present in the blood. As the
nurse you would interpret this blood work as?

A. The patient has an active infection of Hepatitis A.

B. The patient has recovered from a previous Hepatitis A infection and is now immune to it.

C. The patient is in the preicetric phase of viral Hepatitis.

D. The patient is in the icteric phase of viral Hepatitis.

The answer is B. When a patient has anti-HAV (antibodies of the Hepatitis A virus) and IgG, this
means the patient HAD a past infection of Hepatitis A but it is now gone, and the patient is
immune to Hepatitis A now. If the patient had anti-HAV and IgM, this means the patient has an
active infection of Hepatitis A.

8. TRUE or FALSE: A patient with Hepatitis A is contagious about 2 weeks before signs and
symptoms appear and 1-3 weeks after the symptoms appear.

The answer is TRUE.

9. A 25-year-old patient was exposed to the Hepatitis A virus at a local restaurant one week
ago. What education is important to provide to this patient?

A. Inform the patient to notify the physician when signs and symptoms of viral Hepatitis start
to appear.
B. Reassure the patient the chance of acquiring the virus is very low.

C. Inform the patient it is very important to obtain the Hepatitis A vaccine immediately to
prevent infection.

D. Inform the patient to promptly go to the local health department to receive immune
globulin.

The answer is D. Since the patient was exposed to Hepatitis A, the patient would need to take
preventive measures to prevent infection because infection is possible. The patient should not
wait until signs and symptoms appear because the patient can be contagious 2 weeks BEFORE
signs and symptoms appear. The vaccine would not prevent Hepatitis A from this exposure, but
from possible future exposures because it takes the vaccine 30 days to start working. The best
answer is option D. The patient would need to receive immune globulin to provide temporary
immunity within 2 weeks of exposure.

10. Select all the ways a person can become infected with Hepatitis B:

A. Contaminated food/water

B. During the birth process

C. IV drug use

D. Undercooked pork or wild game

E. Hemodialysis

F. Sexual intercourse

The answers are B, C, E, and F. Hepatitis B is spread via blood and body fluids. It could be
transmitted via the birthing process, IV drug use, hemodialysis, or sexual intercourse etc.

11. A patient has completed the Hepatitis B vaccine series. What blood result below would
demonstrate the vaccine series was successful at providing immunity to Hepatitis B?

A. Positive IgG

B. Positive HBsAg

C. Positive IgM

D. Positive anti-HBs

The answer is D. A positive anti-HBs (Hepatitis B surface antibody) indicates either a past
infection of Hepatitis B that is now cleared and the patient is immune, OR that the vaccine has
been successful at providing immunity. A positive HBsAg (Hepatitis B surface antigen) indicates
an active infection.

12. A patient has lab work drawn and it shows a positive HBsAg. What education will you
provide to the patient?

A. Avoid sexual intercourse or intimacy such as kissing until blood work is negative.

B. The patient is now recovered from a previous Hepatitis B infection and is now immune.

C. The patient is not a candidate from antiviral or interferon medications.

D. The patient is less likely to develop a chronic infection.

The answer is A. A positive HBsAg (hepatitis B surface antigen) indicates an active Hepatitis B
infection. Therefore, the patient should avoid sexual intercourse and other forms of intimacy
until their HBsAg is negative.

13. A patient with Hepatitis A asks you about the treatment options for this condition. Your
response is?

A. Antiviral medications

B. Interferon

C. Supportive care

D. Hepatitis A vaccine

The answer is C. There is no current treatment for Hepatitis A but supportive care and rest.
Treatments for the other types of Hepatitis such as B, C, and D include antiviral or interferon
(mainly the chronic cases) along with rest.

14. A patient was exposed to Hepatitis B recently. Postexposure precautions include


vaccination and administration of HBIg (Hepatitis B Immune globulin). HBIg needs to be given
as soon as possible, preferably ___________ after exposure to be effective.

A. 2 weeks

B. 24 hours

C. 1 month

D. 7 days
The answer is B. HBIg should be given 24 hours after exposure to maximum effectiveness of
temporary immunity against Hepatitis B. It would be given within 12 hours after birth to an
infant born to a mother who has Hepatitis B.

15. You’re providing education to a patient with an active Hepatitis B infection. What will you
include in their discharge instructions? Select all that apply:

A. “Take acetaminophen as needed for pain.”

B. “Eat large meals that are spread out through the day.”

C. “Follow a diet low in fat and high in carbs.”

D. “Do not share toothbrushes, razors, utensils, drinking cups, or any other type of personal
hygiene product.”

E. “Perform aerobic exercises daily to maintain strength.”

The answers are C and D. The patient should NOT take acetaminophen (Tylenol) due to its
effective on the liver. The patient should eat small (NOT large), but frequent meals…this may
help with the nausea. The patient should rest (not perform aerobic exercises daily) because this
will help with liver regeneration.

16. What is the MOST common transmission route of Hepatitis C?

A. Blood transfusion

B. Sharps injury

C. Long-term dialysis

D. IV drug use

The answer is D. IV drug use is the MOST common transmission route of Hepatitis C.

17. A patient is diagnosed with Hepatitis D. What statement is true about this type of viral
Hepatitis? Select all that apply:

A. The patient will also have the Hepatitis B virus.

B. Hepatitis D is most common in Southern and Eastern Europe, Mediterranean, and Middle
East.

C. Prevention of Hepatitis D includes handwashing and the Hepatitis D vaccine.

D. Hepatitis D is most commonly transmitted via the fecal-oral route.


The answers are A and B. These are true statements about Hepatitis D. Prevention for Hepatitis
D includes handwashing and the Hepatitis B vaccine (since it occurs only with the Hepatitis B
virus). It is transmitted via blood.

18. Select all the signs and symptoms associated with Hepatitis?

A. Arthralgia

B. Bilirubin 1 mg/dL

C. Ammonia 15 mcg/dL

D. Dark urine

E. Vision changes

F. Yellowing of the sclera

G. Fever

H. Loss of appetite

The answers are A, D, F, G, and H. The bilirubin and ammonia levels are normal in these
options, but they would be abnormal in Hepatitis. A normal bilirubin is 1 or less, and a normal
ammonia is 15-45 mcg/dL.

19. A patient with Hepatitis has a bilirubin of 6 mg/dL. What findings would correlate with this
lab result? Select all that apply:

A. None because this bilirubin level is normal

B. Yellowing of the skin and sclera

C. Clay-colored stools

D. Bluish discoloration on the flanks of the abdomen

E. Dark urine

F. Mental status changes

The answers are B, C, and E. This is associated with a high bilirubin level. A normal bilirubin
level is 1 or less.

20. A patient with Hepatitis is extremely confused. The patient is diagnosed with Hepatic
Encephalopathy. What lab result would correlate with this mental status change?
A. Ammonia 100 mcg/dL

B. Bilirubin 7 mg/dL

C. ALT 56 U/L

D. AST 10 U/L

The answer is A. When ammonia levels become high (normal 15-45 mcg/dL) it affects brain
function. Therefore, the nurse would see mental status changes in a patient with this ammonia
level.

21. The physician writes an order for the administration of Lactulose. What lab result indicates
this medication was successful?

A. Bilirubin <1 mg/dL

B. ALT 8 U/L

C. Ammonia 16 mcg/dL

D. AST 10 U/L

The answer is C. Lactulose is ordered to decrease a high ammonia level. It will cause excretion
of ammonia via the stool. A normal ammonia level would indicate the medication was successful
(normal ammonia level 15-45 mcg/dL).

22. How is Hepatitis E transmitted?

A. Fecal-oral

B. Percutaneous

C. Mucosal

D. Body fluids

The answer is A.

23. Which patient below is at MOST risk for developing a complication related to a Hepatitis E
infection?

A. A 45-year-old male with diabetes.

B. A 26-year-old female in the 3rd trimester of pregnancy.

C. A 12-year-old female with a ventricle septal defect.


D. A 63-year-old male with cardiovascular disease.

The answer is B. Patients who are in the 3rd trimester of pregnancy are at a HIGH risk of
developing a complication related to a Hepatitis E infection.

24. What is the BEST preventive measure to take to help prevent ALL types of viral Hepatitis?

A. Vaccination

B. Proper disposal of needles

C. Hand hygiene

D. Blood and organ donation screening

The answer is C. Hand hygiene can help prevent all types of viral hepatitis. However, not all
types of viral Hepatitis have a vaccine available or are spread through needle sticks or
blood/organs donations. Remember Hepatitis A and E are spread only via fecal-oral routes.

25. Select all the types of viral Hepatitis that have preventive vaccines available in the United
States?

A. Hepatitis A

B. Hepatitis B

C. Hepatitis C

D. Hepatitis D

E. Hepatitis E

The answers are A and B. Currently there is only a vaccine for Hepatitis A and B in the U.S.

26. A patient is prescribed Peginterferon alfa-2a. The nurse will prepare to administer this
medication what route?

A. Oral

B. Intramuscular

C. Subcutaneous

D. Intravenous

The answer is C. This medication is administered subq.


27. A patient with viral Hepatitis states their flu-like symptoms have subsided. However, they
now have yellowing of the skin and sclera along with dark urine. Based on this finding, this is
what phase of Hepatitis?

A. Icteric

B. Posticteric

C. Preicteric

D. Convalescent

The answer is A. The Preicteric (prodromal) Phase: flulike symptoms…joint pain, fatigue,
nausea vomiting, abdominal pain change in taste, liver enzymes and bilirubin
increasing….Icteric Phase: decrease in the flu-like symptoms but will have jaundice and dark
urine (buildup of bilirubin) yellowing of skin and white part of the eyeball, clay-colored stool
(bilirubin not going to stool to give it’s normal brown color) enlarged liver and pain in this
area….Posticteric (convalescent) Phase: jaundice and dark urine start to subside and stool returns
to normal brown color, liver enzymes and bilirubin decrease to normal

28. During the posticteric phase of Hepatitis the nurse would expect to find? Select all that
apply:

A. Increased ALT and AST levels along with an increased bilirubin level

B. Decreased liver enzymes and bilirubin level

C. Flu-like symptoms

D. Resolved jaundice and dark urine

The answers are B and D. Posticteric (convalescent) Phase: jaundice and dark urine start to
subside and stool returns to normal brown color, liver enzymes and bilirubin decrease to normal

You might also like